Download as pdf or txt
Download as pdf or txt
You are on page 1of 39

2 Modern History

PREVIOUS YEARS QUESTIONS – PRELIMS 2011 TO 2022


Modern History : PYQs Analysis
25

20
Modern History

15

10 12 12
9
8 8
7 7
5 6 6 6
5
3
0
2022 2021 2020 2019 2018 2017 2016 2015 2014 2013 2012 2011
* Question count may vary due to subjectivity

Modern History: PYQs Analysis


No. Topic 2022 2021 2020 2019 2018 2017 2016 2015 2014 2013 2012 2011 Total
The Advent of
1 1 1 2
Europeans in India
British Expansion
2 1 1
in India
Economic Policies
3 2 1 2 3 1 3 2 14
of The British
Administrative
4 1 1 1 2 1 1 1 1 2 1 12
Changes After 1857
Peasants and Tribal
5 2 1 1 1 5
Movements
Socio-Religious
6 1 2 2 5
Reforms
Indian National
7 Movement-I 1 1 1 2 1 2 1 9
(1905-1918)
Indian National
8 Movement-II 1 1 1 1 2 1 7
(1918-1929)
Indian National
9 Movement-III 1 2 1 1 1 1 3 1 1 1 2 15
(1930-1947)
Development of
10 1 1 3 5
Education
Prominent
11 Personalities in 2 2 2 2 1 1 1 2 13
Modern History
Constitutional
12 1 1
Developments
Total 3 7 9 6 12 7 6 8 5 6 12 8 89

THE ADVENT OF THE ECONOMIC POLICIES OF THE BRITISH


EUROPEANS IN INDIA 4. With reference to the history of India, consider the
1. With reference to Indian history, consider the following pairs: (2020)
following statements: (2022) 1. Aurang - In-charge of treasury of the State
1. The Dutch established their factories/warehouses 2. Banian - Indian agent of the East India Company
on the east coast on lands granted to them by
3. Mirasidar - Designated revenue payer to the State
Gajapati rulers.
Which of the pairs given above is/are correctly
2. Alfonso de Albuquerque captured Goa from the
Bijapur Sultanate. matched?
3. The English East India Company established a (a) 1 and 2 only
factory at Madras on a plot of land leased from a (b) 2 and 3 only
representative of the Vijayanagara empire. (c) 3 only
Which of the statements given above are correct? (d) 1, 2 and 3
(a) 1 and 2 only
5. Which of the following statements correctly explain
(b) 2 and 3 only
the impact of the Industrial Revolution on India during
(c) 1 and 3 only
the first half of the nineteenth century? (2020)
(d) 1, 2 and 3
(a) Indian handicrafts were ruined.
2. In the first quarter of the seventeenth century, in
(b) Machines were introduced in the Indian textile
which of the following was/were the factory/factories
of the English East India Company located? (2021) industry in large numbers.
1. Broach (c) Railway lines were laid in many parts of the
2. Chicacole country.
3. Trichinopoly (d) Heavy duties were imposed on the imports of
Select the correct answer using the code given British manufactures.
below. 6. Which one of the following groups of plants was
(a) 1 only (b) 1 and 2 domesticated in the ‘New World’ and introduced into
(c) 3 only (d) 2 and 3 the ‘Old World’? (2019)
(a) Tobacco, cocoa and rubber
BRITISH EXPANSION IN INDIA (b) Tobacco, cotton and rubber
3. Which one of the following statements does not apply (c) Cotton, coffee and sugarcane
to the system of Subsidiary Alliance introduced by (d) Rubber, coffee and wheat
Lord Wellesley? (2018)
7. Economically, one of the results of the British rule in
(a) To maintain a large standing army at others
India in the 19th century was the: (2018)
expense
(b) To keep India safe from Napoleonic danger (a) increase in the export of Indian handicrafts
(c) To secure a fixed income for the Company (b) growth in the number of Indian owned factories
(d) To establish British paramountcy over the Indian (c) commercialization of Indian agriculture
States (d) rapid increase in the urban population

40 UPSC Prelims PYQs


8. The staple commodities of export by the English East Which of the statements given above is/are correct?
Indian Company from Bengal in the middle of the (a) 1 only (b) 1 and 2 only
18th century were (2018) (c) 1, 2 and 3 (d) None
(a) Raw cotton, oil-seeds and opium 14. Consider the following statements: (2012)
(b) Sugar, salt, zinc and lead The most effective contribution made by Dadabhai
(c) Copper, silver, gold, spices and tea Naoroji to the cause of Indian National Movement
(d) Cotton, silk, saltpeter and opium was that he-
9. Who among the following was/were associated with 1. Exposed the economic exploitation of India by the
the introduction of Ryotwari Settlement in India British.
during the British Rule? (2017) 2. Interpreted the ancient Indian texts and restored
1. Lord Cornwallis the self-confidence of Indians.
2. Alexander Read 3. Stressed the need for eradication of all the social
3. Thomas Munro evils before anything else.
Select the correct answer using the code given Which of the statements given above is/are correct?
below: (a) 1 only (b) 2 and 3 only
(a) 1 only (b) 1 and 3 only (c) 1 and 3 only (d) 1, 2 and 3
(c) 2 and 3 only (d) 1, 2 and 3 15. Consider the following: (2012)
10. The Trade Disputes Act of 1929 provided for: (2017) 1. Assessment of land revenue on the basis of nature
(a) the participation of workers in the management of the soil and the quality of crops.
of industries. 2. Use of mobile cannons in warfare.
(b) arbitrary powers to the management to quell 3. Cultivation of tobacco and red chillies.
industrial disputes. Which of the above was/were introduced into
(c) an intervention by the British Court in the event India by the English?
of a trade dispute. (a) 1 only (b) 1 and 2
(d) a system of tribunals and a ban on strikes. (c) 2 and 3 (d) None
11. Consider the following statements: (2017) 16. The tendency for increased litigation was visible after
1. The Factories Act, 1881 was passed with a view the introduction of the land settlement system of Lord
to fix the wages of industrial workers and to allow Cornwallis in 1793. The reason for this is normally
the workers to form trade unions. traced to which of the following provisions? (2011)
2. N. M. Lokhande was a pioneer in organizing the (a) Making Zamindar’s position stronger vis-à-vis the
labour movement in British India. ryot
Which of the above statements is/are correct? (b) Making East India Company an overlord of
(a) 1 only (b) 2 only Zamindars.
(c) Both 1 and 2 (d) Neither 1 nor 2 (c) Making judicial system more efficient
12. Who of the following was/were economic critic/ (d) None of the (a), (b) and (c) above.
critics of colonialism in India? (2015) 17. With reference to the period of colonial rule in India,
1. Dadabhai Naoroji “Home Charges” formed an important part of the
2. G. Subramania Iyer drain of wealth from India. Which of the following
3. R.C. Dutt funds constituted “Home Charges”? (2011)
Select the correct answer using the codes given 1. Funds used to support the India Office in London.
below. 2. Funds used to pay salaries and pensions of British
(a) 1 only (b) 1 and 2 only personnel engaged in India.
(c) 2 and 3 only (d) 1, 2 and 3 3. Funds used for waging wars outside India by the
13. With reference to Ryotwari Settlement, consider the British.
following statements: (2012) Select the correct answer using the codes given
1. The rent was paid directly by the peasants to the below:
Government. (a) 1 only
2. The government gave Pattas to the Ryots. (b) 1 and 2 only
3. The lands were surveyed and assessed before (c) 2 and 3 only
being taxed. (d) 1, 2 and 3

Modern History 41
23. The Government of India Act of 1919 clearly defined:
ADMINISTRATIVE CHANGES AFTER 1857 (2015)
18. Which of the following is/are the principal feature(s) (a) The separation of power between the judiciary
of the Government of India Act, 1919? (2012) and the legislature
1. Introduction of diarchy in the executive (b) The jurisdiction of the central and provincial
government of the provinces. governments
2. Introduction of separate communal electorates (c) The powers of the Secretary of State for India and
for Muslims. the Viceroy
3. Devolution of legislative authority by the centre (d) None of the above
to the provinces. 24. The Montague-Chelmsford Proposals were related
Select the correct answer using the codes given to: (2016)
below: (a) social reforms
(a) 1 only (b) 2 and 3 only (b) educational reforms
(c) 1 and 3 only (d) 1, 2 and 3 (c) reforms in police administration
19. The distribution of powers between the Centre and (d) constitutional reforms
the States in the Indian Constitution is based on the 25. In the context of Indian history, the principle of
scheme provided in the: (2012) ‘dyarchy (diarchy)’ refers to: (2017)
(a) Morley-Minto Reforms, 1909 (a) division of the central Legislature into two houses.
(b) Montagu-Chelmsford Act, 1919 (b) introductions of double Government i.e., central
(c) Government of India Act, 1935 and Statement governments.
(d) Indian Independence Act, 1947 (c) having two sets of rulers; One in London and
20. What was the purpose for which Sir William another in Delhi.
Wedderburn and W.S. Caine had set up the Indian (d) division of the subjects delegated to the provinces
Parliamentary Committee in 1893? (2011) into two categories.
(a) To agitate for Indian political reforms in the House 26. The object of the Butler Committee of 1927 was to:
of Commons (2017)
(b) To campaign for the entry of Indians into the (a) Define the jurisdiction of the Central and
Imperial Judiciary Provincial Governments.
(c) To facilitate a discussion on India’s Independence (b) Define the powers of the Secretary of State for
in the British Parliament India.
(d) To agitate for the entry of eminent Indians into (c) Impose censorship on national press.
the British Parliament (d) Improve the relationship between the Government
21. The Ilbert Bill controversy was related to the: (2013) of India and the Indian States.
(a) Imposition of certain restrictions to carry arms 27. In the Federation established by the Government of
by the Indians India Act of 1935, residuary powers were given to
(b) Imposition of restrictions on newspapers and the: (2018)
magazines published in Indian languages (a) Federal Legislature
(c) Removal of disqualifications imposed on the (b) Governor General
Indian magistrates with regard to the trial of the (c) Provincial Legislature
Europeans (d) Provincial Governors
(d) Removal of a duty on imported cotton cloth 28. Consider the following statements about ‘the Charter
22. What was/were the object/objects of Queen Victoria’s Act of 1813’: (2019)
Proclamation (1858)? (2014) 1. It ended the trade monopoly of the East India
1. To disclaim any intention to annex the Indian Company in India except for trade in tea and trade
States with China.
2. To place the Indian administration under the 2. It asserted the sovereignty of the British Crown
British Crown. over the Indian territories held by the Company.
3. To regulate East India Company’s trade with India 3. The revenues of India were now controlled by the
Select the correct answer using the code given British Parliament.
below. Which of the statements given above are correct?
(a) 1 and 2 only (b) 2 only (a) 1 and 2 only (b) 2 and 3 only
(c) 1 and 3 only (d) 1, 2 and 3 (c) 1 and 3 only (d) 1, 2 and 3

42 UPSC Prelims PYQs


29. Consider the following statements: (2021) 34. With reference to the history of India, “Ulgulan” or
1. The Montagu-Chelmsford Reforms of 1919 the Great Tumult is the description of which of the
recommended granting voting rights to all the following events? (2020)
women above the age of 21. (a) The Revolt of 1857
2. The Government of India Act of 1935 gave women (b) The Mappila Rebellion of 1921
reserved seats in the legislature. (c) The Indigo Revolt of 1859 – 60
Which of the statements given above is/are correct? (d) Birsa Munda’s Revolt of 1899-1900
(a) 1 only (b) 2 only
(c) Both 1 and 2 (d) Neither 1 nor 2 SOCIO-RELIGIOUS REFORMS
PEASANT AND TRIBAL MOVEMENT 35. During the Indian freedom struggle, the National
Social Conference was formed. What was the reason
30. Which amongst the following provided a common for its formation? (2012)
factor for tribal insurrection in India in the 19th
(a) Different social reform groups or organizations
century? (2011)
of the Bengal region united to form a single body
(a) Introduction of a new system of land revenue and
to discuss the issues of larger interest and to
taxation of tribal products
prepare appropriate petitions/representations
(b) Influence of foreign religious missionaries in tribal to the government.
areas
(b) Indian National Congress did not want to include
(c) Rise of a large number of money lenders, traders
social reforms in its deliberations and decided to
and revenue farmers as middlemen in tribal areas
form a separate body for such a purpose.
(d) The complete disruption of the old agrarian order
(c) Behramji Malabari and M.G. Ranade decided to
of the tribal communities
bring together all the social reform groups of the
31. The demand for the Tebhaga Peasant Movement in country under one organization.
Bengal was for: (2013)
(d) None of the statements (a), (b) and (c) given above
(a) the reduction of the share of the landlords from is correct in this context.
one-half of the crop to one third.
36. Which of the following statements is/are correct
(b) the grant of ownership of land to peasants as they
regarding Brahmo Samaj? (2012)
were the actual cultivators of the land.
(c) the uprooting of Zamindari system and the end 1. It opposed idolatry.
of serfdom. 2. It denied the need for a priestly class for
(d) writing off all peasant debts. interpreting the religious texts.
3. It popularized the doctrine that the Vedas are
32. After the Santhal Uprising subsided, what was/
were the measure/measures taken by the colonial infallible.
government? (2018) Select the correct answer using the codes given
1. The territories called ‘Santhal Paraganas’ were below:
created. (a) 1 only (b) 1 and 2 only
2. It became illegal for a Santhal to transfer land to (c) 3 only (d) 1, 2 and 3
a non-Santhal. 37. Satya Shodhak Samaj organized: (2016)
Select the correct answer using the code given (a) a movement for upliftment of tribals in Bihar
below: (b) a temple-entry movement in Gujarat
(a) 1 only (b) 2 only
(c) an anti-caste movement in Maharashtra
(c) Both 1 and 2 (d) Neither 1 nor 2
(d) a peasant movement in Punjab
33. Indigo cultivation in India declined by the beginning
of the 20th century because of (2020) 38. Consider the following: (2016)
(a) peasant resistance to the oppressive conduct of 1. Calcutta Unitarian Committee
planters 2. Tabernacle of New Dispensation
(b) its unprofitability in the world market because of 3. Indian Reform Association
new inventions Keshab Chandra Sen is associated with the
(c) national leaders’ opposition to the cultivation of establishment of which of the above?
indigo (a) 1 and 3 only (b) 2 and 3 only
(d) Government control over the planters (c) 3 only (d) 1, 2 and 3

Modern History 43
39. In the context of Indian history, the Rakhmabai case 44. Which one of the following movements has contributed
of 1884 revolved around: (2020) to a split in the Indian National Congress resulting
1. women’s right to gain education in the emergence of ‘moderates’ and ‘extremists’?
2. age of consent (2015)
3. restitution of conjugal rights (a) Swadeshi Movement
Select the correct answer using the code given (b) Quit India Movement
below:
(c) Non-Cooperation Movement
(a) 1 and 2 only
(b) 2 and 3 only (d) Civil Disobedience Movement
(c) 1 and 3 only 45. What was the main reason for the split in the Indian
(d) 1, 2 and 3 National Congress at Surat in 1907? (2016)
(a) Introduction of communalism into Indian politics
INDIAN NATIONAL MOVEMENT–I by Lord Minto
(1905-1918) (b) Extremists’ lack of faith in the capacity of
the moderates to negotiate with the British
40. Consider the following freedom fighters: (2022)
Government
1. Barindra Kumar Ghosh
(c) Foundation of Muslim League
2. Jogesh Chandra Chatterjee
3. Rash Behari Bose (d) Aurobindo Ghosh’s inability to the elected as the
Who of the above was/were actively associated President of the Indian National Congress
with the Ghadar Party? 46. The ‘Swadeshi’ and ‘Boycott’ were adopted as methods
(a) 1 and 2 (b) 2 only of struggle for the first time during the: (2016)
(c) 1 and 3 (d) 3 only (a) agitation against the Partition of Bengal
41. What was the reason for Mahatma Gandhi to organise (b) Home Rule Movement
a satyagraha on behalf of the peasants of Kheda?
(c) Non-Cooperation Movement
(2011)
1. The Administration did not suspend the land (d) visit of the Simon Commission to India
revenue collection in spite of a drought. 47. Which one of the following is a very significant aspect
2. The Administration proposed to introduce of the Champaran Satyagraha? (2018)
Permanent Settlement in Gujarat. (a) Active all-India participation of lawyers, students
Which of the statements given above is/are correct? and women in the National Movement
(a) 1 only (b) 2 only
(b) Active involvement of Dalit and Tribal communities
(c) Both 1 and 2 (d) Neither 1 nor 2
of India in the National Movement
42. The Partition of Bengal made by Lord Curzon in 1905 (c) Joining of peasant unrest to India’s National
lasted until: (2014)
Movement
(a) The First World War when Indian troops were
needed by the British and the partition was ended. (d) Drastic decrease in the cultivation of plantation
crops and commercial crops
(b) King George V abrogated Curzon’s Act as the Royal
Durbar in Delhi in 1911 48. With reference to Swadeshi Movement consider the
(c) Gandhiji launched his Civil Disobedience following statements: (2019)
Movement. 1. It contributed to the revival of the indigenous
(d) The Partition of India in 1947 when East Bengal artisan crafts and industries.
became East Pakistan.
2. The National Council of Education was established
43. The Ghadr (Ghadar) was a: (2014) as a part of the Swadeshi Movement.
(a) revolutionary association of Indians with Which of the statements given above is/are correct?
headquarters at San Francisco.
(a) 1 only
(b) nationalist organisation operating from Singapore.
(b) 2 only
(c) militant organisation with headquarters at Berlin
(c) Both 1 and 2
(d) Communist movement for India’s freedom with
head-quarters at Tashkent. (d) Neither 1 nor 2

44 UPSC Prelims PYQs


54. With reference to Rowlatt Satyagraha, which of the
INDIAN NATIONAL MOVEMENT–II following statements is/are correct ? (2015)
(1918-1929) 1. The Rowlatt Act was based on the recommen-
dations of the ‘Sedition Committee’.
49. With reference to the period of Indian freedom struggle,
which of the following was/were recommended by 2. In Rowlatt Satyagraha, Gandhiji tried to utilise the
the Nehru report? (2011) Home Rule League.
1. Complete Independence for India. 3. Demonstrations against the arrival of the Simon
2. Joint electorates for reservation of seats for Commission coincided with Rowlatt Satyagraha.
minorities Select the correct answer using the codes given
3. Provision of fundamental right for the people of below.
India in the constitution. (a) 1 only
Select the correct answer using the codes given (b) 1 and 2 only
below: (c) 2 and 3 only
(a) 1 only (b) 2 and 3 only
(d) 1, 2 and 3
(c) 1 and 3 only (d) 1, 2 and 3
55. In 1920, which of the following changed its name to
50. The Rowlatt Act aimed at: (2012) “Swarajya Sabha”? (2018)
(a) Compulsory economic support to war efforts (a) All India Home Rule League
(b) Imprisonment without trial and summary
(b) Hindu Mahasabha
procedures for trial
(c) South Indian Liberal Federation
(c) Suppression of the Khilafat Movement.
(d) Imposition of restrictions on freedom of the press. (d) The Servants of India Society

51. The Lahore Session of the Indian National Congress


(1929) is very important in history, because: (2012)
INDIAN NATIONAL MOVEMENT - III
1. The Congress passed a resolution demanding (1930-1947)
complete independence.
56. With reference to the proposals of Cripps Mission,
2. The rift between the extremists and moderates consider the following statements: (2022)
was resolved in that Session.
1. The Constituent Assembly would have members
3. A resolution was passed rejecting the two-nation
nominated by the Provincial Assemblies as well
theory in that Session
as the Princely States.
Which of the statements given above is/are correct?
(a) 1 only (b) 2 and 3 2. Any Province, which is not prepared to accept the
new Constitution would have the right to sign a
(c) 1 and 3 (d) None of the above
separate agreement with Britain regarding its
52. The people of India agitated against the arrival of the future status.
Simon Commission because: (2013)
Which of the statements given above is/are correct?
(a) Indians never wanted the review of the working (a) 1 only (b) 2 only
of the Act of 1919
(c) Both 1 and 2 (d) Neither 1 nor 2
(b) Simon Commission recommended the abolition
of Dyarchy (Diarchy) in the Provinces 57. Which one of the following observations is not true
(c) There was no Indian member in the Simon about the Quit India Movement of 1942? (2011)
Commission (a) It was a non-violent movement
(d) The Simon Commission suggested the partition (b) It was led by Mahatma Gandhi
of the country (c) It was a spontaneous movement
53. The 1929 Session of Indian National Congress is on (d) It did not attract the labour class in general
significance in the history of the Freedom Movement
58. With reference to Indian freedom struggle, Usha
because the: (2014)
Mehta is well-known for: (2011)
(a) attainment of Self-Government was declared as
the objective of the Congress. (a) Running the secret Congress Radio in the wake of
Quit India Movement
(b) Attainment of Poorna Swaraj was adopted as the
goal of the Congress. (b) Participating in the second round Table Conference
(c) Non-Cooperation Movement was launched. (c) Leading a contingent of Indian National Army
(d) Decision to participate in the Round Table (d) Assisting in the formation of Interim Government
Conference in London was taken. under Pandit Jawaharlal Nehru

Modern History 45
59. The Congress ministries resigned in the seven 65. The plan of Sir Stafford Cripps envisaged that after
provinces in 1939, because: (2012) the Second World War: (2016)
(a) The Congress could not form ministries in the (a) India should be granted complete independence
other four provinces. (b) India should be partitioned into two before
(b) Emergence of a ‘left wing’ in the Congress made granting independence
the working of the ministries impossible. (c) India should be made a republic with the condition
(c) There were widespread communal disturbances that she will join the Commonwealth
in their provinces. (d) India should be given Dominion status
(d) None of the statements (a), (b) and (c) given above 66. With reference to Indian freedom struggle, consider
is correct. the following events: (2017)
60. Quit India Movement was launched in response to: 1. Mutiny in Royal Indian Navy
(2013) 2. Quit India Movement launched
(a) Cabinet Mission Plan 3. Second Round Table Conference
(b) Cripps Proposals What is the correct chronological sequence of the
(c) Simon Commission Report above events?
(d) Wavell Plan (a) 1-2-3 (b) 2-1-3
61. The Radcliffe Committee was appointed to: (2014) (c) 3-2-1 (d) 3-1-2
(a) Solve the problem of minorities in India 67. With reference to the British colonial rule in India,
(b) Given effect to the Independence Bill consider the following statements: (2019)
(c) Delimit the boundaries between India and 1. Mahatma Gandhi was instrumental in the
Pakistan abolition of the system of ‘indentured labour’.
(d) Enquire into the riots in East Bengal 2. In Lord Chelmsford’s War Conference’, Mahatma
62. With reference to Congress Socialist Party, consider Gandhi did not support the resolution on
the following statements: (2015) recruiting Indians for World War.
1. It advocated the boycott of British goods and 3. Consequent upon the breaking of the Salt Law by
evasion of taxes. the Indian people, the Indian National Congress
2. It wanted to establish the dictatorship of the was declared illegal by the colonial rulers.
proletariat. Which of the statements given above are correct?
3. It advocated separate electorate for minorities (a) 1 and 2 only
and oppressed classes. (b) 1 and 3 only
Which of the statements given above is/are correct? (c) 2 and 3 only
(a) 1 and 2 only (b) 3 only (d) 1, 2 and 3
(c) 1, 2 and 3 (d) None
68. The Gandhi-Irwin Pact included which of the
63. Who of the following organised a march on the Tanjore following? (2020)
coast to break the Salt Law in April 1930? (2015) 1. Invitation to Congress to participate in the Round
(a) V.O. Chidambaram Pillai Table Conference
(b) C. Rajagopalachari 2. Withdrawal of Ordinances promulgated in
(c) K. Kamaraj connection with the Civil Disobedience Movement
(d) Annie Besant 3. Acceptance of Gandhi’s suggestion for enquiry
64. With reference to the Cabinet Mission, which of the into police excesses.
following statements is/are correct? (2015) 4. Release of only those prisoners who were not
1. It recommended a federal government. charged with violence.
2. It enlarged the powers of the Indian courts. Select the correct answer using the code given
3. It provided for more Indians in the ICS. below:
(a) 1 only
Select the correct answer using the code given
below. (b) 1, 2 and 4 only
(a) 1 only (b) 2 and 3 (c) 3 only
(c) 1 and 3 (d) None of these (d) 2, 3 and 4 only

46 UPSC Prelims PYQs


69. With reference to 8th August 1942 in Indian history, 74. Wellesley established the Fort William College at
which one of the following statements is correct? Calcutta because: (2020)
(2021) (a) he was asked by the Board of Directors at London
(a) The Quit India Resolution was adopted by the to do so
AICC. (b) he wanted to revive interest in oriental learning
(b) The Viceroy’s Executive Council was expanded to in India
include more Indians. (c) he wanted to provide William Carey and his
(c) The Congress ministries resigned in seven associates with employment
provinces. (d) he wanted to train British civilians for
(d) Cripps proposed an Indian Union with full administrative purpose in India
Dominion Status once the Second World War was 75. Who among the following was associated as Secretary
over. with Hindu Female School which later came to be
70. In the context of Colonial India, Shah Nawaz Khan, known as Bethune Female School? (2021)
Prem Kumar Sehgal and Gurbaksh Singh Dhillon are (a) Annie Besant
remembered as: (2021) (b) Debendranath Tagore
(a) leaders of Swadeshi and Boycott Movement (c) Ishwar Chandra Vidyasagar
(b) members of the Interim Government in 1946 (d) Sarojini Naidu
(c) members of the Drafting Committee in the
Constituent Assembly PROMINENT PERSONALITIES IN
(d) officers of the Indian National Army MODERN HISTORY
DEVELOPMENT OF EDUCATION 76. Which of the following parties were established by
Dr. B.R. Ambedkar? (2012)
71. Regarding Wood’s Dispatch, which of the following 1. The Peasants and Workers Party of India.
statements are true? (2018) 2. All India Scheduled Castes Federation.
1. Grants-in-Aid system was introduced. 3. The Independent Labour Party.
2. Establishment of universities was recommended. Select the correct answer using the codes given
3. English as a medium of instruction at all levels of below:
education was recommended. (a) 1 and 2 only (b) 2 and 3 only
Select the correct answer using the code given (c) 1 and 3 only (d) 1, 2 and 3
below: 77. Mahatma Gandhi undertook fast unto death in 1932,
(a) 1 and 2 only (b) 2 and 3 only mainly because: (2012)
(c) 1 and 3 only (d) 1, 2 and 3 (a) Round Table Conference failed to satisfy Indian
72. Which of the following led to the introduction of political aspirations.
English Education in India? (2018) (b) Congress and Muslim League had differences of
1. Charter Act of 1813 opinion.
2. General Committee of Public Instruction, 1823 (c) Ramsay Macdonald announced the Communal
3. Orientalist and Anglicist Controversy. Award.
Select the correct answer using the code given (d) None of the statements (a), (b) and (c) given above
below: is correct in this context.
(a) 1 and 2 only (b) 2 only 78. Annie Besant was: (2013)
(c) 1 and 3 only (d) 1, 2 and 3 1. Responsible for starting the Home Rule Movement
73. With reference to educational institutions during 2. The founder of the Theosophical Society
colonial rule in India, consider the following pairs: 3. Once the President of the Indian National
Institution Founder: (2018) Congress
1. Sanskrit College at Benaras - William Jones Select the correct statement/statements using the
2. Calcutta Madrasa - Warren Hastings codes given below.
3. Fort William College - Arthur Wellesley (a) 1 only
Which of the pairs given above is/are correct? (b) 2 and 3 only
(a) 1 and 2 (b) 2 only (c) 1 and 3 only
(c) 1 and 3 (d) 3 only (d) 1, 2 and 3

Modern History 47
79. Consider the following statements: (2015) 3. P. C. Joshi : General Secretary,
1. The first woman President of the Indian National Communist Party of
Congress was Sarojini Naidu. India
2. The first Muslim President of the Indian National Which of the pairs given above is/are correctly matched?
Congress was Badruddin Tyabji. (a) 1 only (b) 1 and 2 only
Which of the statements given above is/are correct? (c) 3 only (d) 1, 2 and 3
(a) 1 only (b) 2 only
85. With reference to the book “Desher Katha” written
(c) Both 1 and 2 (d) Neither 1 nor 2
by Sakharam Ganesh Deuskar during the freedom
80. Consider the following pairs: (2017) struggle, consider the following statement: (2020)
1. Radhakanta Deb – first President of the British 1. It warned against the Colonial States hypnotic
Indian Association conquest of the mind.
2. Gazulu Lakshminarasu Chetty – founder of the 2. It inspired the performance of swadeshi street
Madras Mahajan Sabha plays and folk songs.
3. Surendranath Banerjee – Founder of the Indian 3. The use of ‘desh’ by Deuskar was in the specific
Association context of the region of Bengal.
Which of the above pairs is/are correctly matched? Which of the statements given above are correct?
(a) 1 only (b) 1 and 3 only (a) 1 and 2 only (b) 2 and 3 only
(c) 2 and 3 only (d) 1, 2 and 3 (c) 1 and 3 only (d) 1, 2 and 3
81. He wrote biographies of Mazzini, Garibaldi, Shivaji 86. The Vital-Vidhvansak, the first monthly journal to
and Shrikrishna; stayed in America for some time, have the untouchable people as its target audience
and was also elected to the Central Assembly. He was was published by: (2020)
(2018) (a) Gopal Baba Walangkar
(a) Aurobindo Ghosh (b) Bipin Chandra Pal (b) Jyotiba Phule
(c) Lala Lajpat Rai (d) Motilal Nehru (c) Mohandas Karamchand Gandhi
82. Which among the following events happened earliest? (d) Bhimrao Ramji Ambedkar
(2018) 87. With reference to Madanapalle of Andhra Pradesh,
(a) Swami Dayanand established Arya Samaj. which one of the following statements is correct?
(b) Dinabandhu Mitra wrote Neeldarpan. (2021)
(c) Bankim Chandra Chattopadhyay wrote Anandmath. (a) Pingali Venkayya designed the tricolour Indian
(d) Satyendranath Tagore became the first Indian to National Flag here.
succeed in the Indian Civil Services Examination. (b) Pattabhi Sitaramaiah led the Quit India Movement
of Andhra region from here.
83. Consider the following pairs: (2019)
(c) Rabindranath Tagore translated the National
Movement : Organization Leader Anthem from Bengali to English here.
1. All India Anti- : Mahatma Gandhi (d) Madame Blavatsky and Colonel Olcott set up
headquarters of Theosophical Society first here.
Untouchability
League 88. Who among the following is associated with ‘Song
from Prison’, a translation of ancient Indian religious
2. All India Kisan : Swami Sahajanand lyrics in English? (2021)
Sabha Saraswati (a) Bal Gangadhar Tilak
3. Self-Respect : E. V. Ramaswami (b) Jawaharlal Nehru
Movement Naicker (c) Mohandas Karamchand Gandhi
Which of the pairs given above is/are correctly (d) Sarojini Naidu
matched?
(a) 1 only (b) 1 and 2 only CONSTITUTIONAL DEVELOPMENT
(c) 2 and 3 only (d) 1, 2 and 3
89. With reference to Indian History, the Members of the
84. With reference to Indian National Movement, consider
Constituent Assembly from the Provinces were:
the following pairs: (2019)
(2013)
Person : Position held (a) directly elected by the people of those Provinces
1. Sir Tej Bahadur : President, All India (b) nominated by the Indian National Congress and
the Muslim League
Sapru Liberal Federation
(c) elected by the Provincial Legislative Assemblies
2. K. C. Neogy : Member, The (d) selected by the Government for their expertise in
Constituent Assembly constitutional matters

48 UPSC Prelims PYQs


Answer Key
1. (b) 2. (a) 3. (c) 4. (b) 5. (a) 6. (a) 7. (c) 8. (d) 9. (c) 10. (d)
11. (b) 12. (d) 13. (c) 14. (a) 15. (d) 16. (d) 17. (b) 18. (c) 19. (c) 20. (a)
21. (c) 22. (a) 23. (b) 24. (d) 25. (d) 26. (d) 27. (b) 28. (a) 29. (b) 30. (d)
31. (a) 32. (c) 33. (b) 34. (d) 35. (b) 36. (b) 37. (c) 38. (b) 39. (b) 40. (d)
41. (a) 42. (b) 43. (a) 44. (a) 45. (b) 46. (a) 47. (c) 48. (c) 49. (b) 50. (b)
51. (a) 52. (c) 53. (b) 54. (b) 55. (a) 56. (b) 57. (b) 58. (a) 59. (d) 60. (b)
61. (c) 62. (d) 63. (b) 64. (a) 65. (d) 66. (c) 67. (b) 68. (b) 69. (a) 70. (d)
71. (a) 72. (d) 73. (b) 74. (d) 75. (c) 76. (b) 77. (c) 78. (c) 79. (b) 80. (b)
81. (c) 82. (b) 83. (d) 84. (d) 85. (a) 86. (a) 87. (c) 88. (c) 89. (c)

Explanation
1. (b) Sub-Theme: Various events during the advent of and council of the Surat factory which was
the Europeans established in the year 1612.
Statement 1 is incorrect: There is a difference z South-eastern Coast: EIC established their
in timeline of the events. The end of Gajapati factories at Masulipatam in 1611 and
Dynasty was in 1542 and the United East India Armagaon near Pulicat in 1626.
Company of the Netherlands, formed in March z Eastern India: In eastern India EIC
1605 by the Charter of Dutch Parliament and established their factories at Hariharpur
founded their first factory in Masulipatnam in and Balasore(now Odisha) in 1633, at
Andhra in 1605. Hugli in 1651, followed by setting up
Statement 2 is correct: Alfonso de Albuquerque factories at Patna, Dacca, Kasimbazar in
considered as Founder of the Portuguese Bengal and Bihar respectively.
power in India: captured Goa from Bijapur;
persecuted Muslims; captured Bhatkal from Sri NOTE: This is a direct question asked
Krishna Deva Rai (1510) of Vijayanagara. from Spectrum Modern India (revised
edition)
Statement 3 is correct: The English East India
Company established a factory in Madras in 3. (c) Sub-Theme: British Expansionist Policy
1639 on land leased from representatives of The system aimed to establish British
Vijayanagara Empire called the Nayakas. paramountcy over the Indian States and to
2. (a) Sub-Theme: Factories of English East India ensure their military and political subservience
Company to the British East India Company. Securing a
The English East India Company had a factory fixed income for the company was not the
at Broach (present-day Bharuch) in Gujarat, primary objective of the system.
India during the seventeenth century. However, Subsidiary Alliance by Wellesley (1798-
Chicacole (present-day Srikakulam) and 1805):
Trichinopoly (present-day Tiruchirappalli) z Allying Indian states maintained the British
were not among the locations of the Company’s army at its frontier and paid for their
factories during this period. maintenance. Ruler had to post British
Establishment of Factories by English East officials at their court. Ruler couldn’t employ
India Company: Europeans without the British’s permission.
z West Coast: The English India Company Couldn’t contact enemies to make peace.
established their factories at Agra, z Maintenance asked was high → Ruler’s
Ahmadabad, Baroda and Broach by couldn’t pay → asked to cede part of their
1619, under the control of the president territory.

Modern History 49
z Awadh was first to come under it. 5. (a) Sub-Theme: Impact of Industrial Revolution on
z Signed by: Nawab of Awadh, Nizam of indigenous industry
Hyderabad, Ruler of Mysore, Ruler of Declining and deindustrialisation of the Indian
Tanjore, Peshwa, Bhonsle of Berar, The traditional cottage and handicraft industry was
Sindhias, The Rajput states, The ruler of a direct effect of the industrial revolution in
Bharatpur, Holkars. England along with the colonisation of India.
z During this phase, almost all the Indian Industrial Revolution and its impact on
States came under British subordination, India’s indigenous industry
by compelling them to enter into subsidiary z The Industrial Revolution brought severe
alliances with them. Also the Indian Princes consequences to Indian society.
were put under obligation to accept the z The fall and destruction of the urban and
British Paramountcy. rural handicraft industry was one of the
z It was a ‘Non-Intervention Policy’ used by most significant effects of British Rule..
Lord Wellesley to bring the Princely states z The Industrial Revolution in England also
under the control of the British. affected the textile industries in India in
z Though the native Indian ruler had to pay many ways, such as:
for the maintenance of the British army and  English textiles were now posing a
if he failed to make the payment, a portion serious threat to Indian textiles in the
of his territory was taken away and ceded European and American markets.
to the British, but securing fixed income  Textiles from India were subject to high
was never part of the system of Subsidiary taxes in Britain.
Alliance.
 The African markets were successfully
NOTE: Always stay focused and calm while taken over by cheap, machine-made
solving this type of question, often in this type textiles from Britain.
of question we miss/overlook the negative  By the 1830s, inexpensive British
keyword such as ‘Does Not’/’Not’/’Not manufactured products had inundated
Correct’, etc. in the question and mark the the Indian marketplaces. The Indian
wrong answer in a haste. textile industries suffered as a result of
4. (b) Sub-Theme: Important Keyword related to them being less expensive than Indian
Colonial Revenue Policy textiles.
Pair 1 is incorrect: Aurang is a Persian term z Following the Industrial Revolution, Britain
referring to a warehouse where goods are began making textiles by a machine that
collected before being sold. were considerably more affordable and of
Pair 2 is correct: The word Bania (also Vania) higher quality than conventional Indian
is derived from the Sanskrit vanij, meaning textiles.
‘a merchant’. The banian acted as an agent z The start of the industrial revolution had
for individual East India Company managers a negative impact on the Indian handicraft
and performed a range of functions including, industry.
bookkeeping, interpreting, and brokerage,
z Additionally, the decline of the kings,
as well as managing his household and
chieftains, and zamindars who served as the
personal business activities. Banians served as
employers of these masters of handicrafts led
middlemen for European merchants in Bengal
to the steady expansion of British dominion
during the British era.
in India through outright battles with the
Pair 3 is correct: Mirasdar in Urdu means
rulers, which resulted in the suffering of
hereditary landowner, co-proprietor and in
handicrafts in India.
Persian, miras is inheritance, estate. Under
the Ryotwari settlement system in Southern z India became both a raw material supplier
India, the East India Company recognized and a finished goods market. India, a country
mirasdars as the only proprietors of land, based on agriculture, became an economic
dismissing tenants’ rights completely. They colony.
were prohibited by law from selling their land z The British items were inexpensive and only
but it can be leased to tenants for a nominal required a little amount of duty, while the
fee in return for its cultivation. They acted as a Indian goods were subject to heavy import
designated revenue payer of the state. duties.

50 UPSC Prelims PYQs


NOTE: The question is talking about the “first wheat, barley, pulses, peas, rice, sesame,
half of the nineteenth century”. We all linseed, and mustard and traces of cotton
know, the first railway in India was started in some places were also found. However
in 1853, between Bori Bunder (Bombay) tobacco, cocoa and rubber came to India
and Thane. Moreover, the Bombay Spinning through Europeans in the late medieval
and Weaving Company was the first cotton or early modern era. So from our basic
mill to be established in Bombay in 1854 by knowledge we can eliminate the wrong
Cowaszee Nanabhoy Davar and his associates. options and reach the correct answer.
This factual knowledge helps us to eliminate
7. (c) Sub-Theme: Commercialisation of Indian
statements (b) and (c). Statement (d) is itself
Agriculture
contradictory to the nature of British trade and
economy. Why would Britishers discourage One of the results of the British rule in India
the profitability of goods coming from British in the 19th century was the commercialization
manufacturers? This is highly unlikely !! of agriculture by introducing cash crops like
6. (a) Sub-Theme: Columbian Exchange opium and indigo
z Colonial rule has negatively impacted the
Tobacco, cocoa, and rubber are the groups of
plants that were domesticated in the ‘New Indian industries and handicrafts.
World’ and introduced into the ‘Old World’ z The increase in the import of finished
through the European explorers in the late manufactured goods poses a threat to the
medieval or early modern era. indigenous factories.
The Columbian Exchange: z It was, economically, one of the results of
z The Columbian exchange, also known British rule in India in the 19th Century
as the Columbian interchange, named for was the growth of commercial crops like
Christopher Columbus, was the widespread cotton, jute, groundnut, oilseeds, tobacco,
transfer of plants, animals, culture, human
sugarcane, etc. and the raw materials for
populations, technology, diseases, and ideas
the industries in Britain.
between the Americas, West Africa, and the
Old World in the 15th and 16th centuries. z There was no rapid increase in the urban
z It also relates to European colonization and population during 19th Century British
trade following Christopher Columbus’s India.
1492 voyage. Invasive species, including NOTE: If we read NCERT/Spectrum
communicable diseases, were a by-product carefully, we can easily observe a pattern in
of the Exchange. given sentences: in sentence 1, “decrease”
z Old World to New World: Coffee, wheat, replaced by “increase”, in sentence 2,
cotton, barley, rice, sugarcane and sugar “foreign” replaced by “Indian” and in
beet, etc. sentence 3, “rural” is replaced by “urban”.
z New World to Old World: avocado, cashew, 8. (d) Sub-Theme: Colonial Trade
cocoa bean, potato, maize, rubber, tobacco, etc.
The East India Company was originally formed
NOTE: Even if we don’t know the exact in Britain for pursuing trade with the East
answer of this particular question, we can Indies in Southeast Asia. In fact, it ended up
still attempt this question with our basic trading mainly with the Indian subcontinent
knowledge. For instance, Cotton and wheat and China, where the main items of trade
has been a widely cultivated crop in India were cotton, silk, tea, opium, and saltpetre
since ancient times. Even Harappans grew (potassium nitrate).

9. (c) Sub-Theme: Land Revenue Settlement

z Started by Lord Cornwallis in 1793.


z Prevalent in Bengal, Bihar, Orissa, Varanasi (United Provinces), and Northern
Permanent Karnataka.
Settlement z Covered 19% of British India.
z A new section of Zamindars was appointed who would provide 10/11th part of the
revenue collected to the British while keeping the remaining.

Modern History 51
z Started by Thomas Munro and Alexander Reed in 1820.
z Started in the Baramahal district of Tamil Nadu and later spread to Madras, parts of
Bombay, East Bengal, Assam and Kurg (Karnataka).
Ryotwari
z Covered 51% of British India.
System
z Ryots (farmers) were given ownership and other rights over the land and were
required to pay the revenue directly to the government. Involved high taxes and
strict collection.
z The Mahalwari system was introduced by Holt Mackenzie in 1822.
z Prevalent in United Provinces, Central Provinces and Punjab.
Mahalwari z Covered 30% of British India.
System z The village or mahal served as the unit for collecting revenue. Village land belonged
to the community and it was the responsibility of the entire community to pay the
land revenue.

of children under the age 7. Children between


NOTE: Questions on the settlement system
earlier came in 2011 and two questions in the ages 7-12 were to work for a maximum 9
2012. Reference of PYQs and research about hours. It also made compulsory that dangerous
given options stands important. machinery should be fenced properly, but no
mention of forming trade unions.
10. (d) Sub-Theme: Labour Legislations Statement 2 is correct: Narayan Meghaji
About the Trade Disputes Act, 1929: Lokhande (1848–1897) was the father of
z The Trade Disputes Act, 1929 was codified the trade union movement in India. He is
for five years as an experimental measure. remembered not only for ameliorating the
z The main objective of the Act was to make working conditions of textile mill-hands in
provisions for the establishment of Courts the 19th century but also for his courageous
of Inquiry and Boards of Conciliation with a initiatives on caste and communal issues.
view to investigate and settle trade disputes. Lokhande is acclaimed as the Father of the
z The Act banned unnoticed strikes or lockouts Trade Union Movement in India. Some of
in the provision of public utility services. the rights mill workers got because of N M
z A strike or lockout that was intended to Lokhande were:
cause severe, widespread, or prolonged z Mill workers should get a weekly holiday on
hardship for the community and compel Sunday
the government to take or refrain from z In the afternoon, workers should be entitled
taking a particular course of action was also
to half-hour recess.
made illegal.
z The mill should start working from 6:30 in
z This included strikes and lockouts that had
the morning and close by sunset.
any purpose other than to advance a trade
dispute within the trade or industry in z The salaries of the workers should be given
which the strikers or the employers locking by the 15th of every month.
out were engaged. NOTE: From NCERT, we know that N.M.
z The Act was amended in 1932 and was Lokhande was a pioneer of the labour
made permanent by the Trade Disputes movement in India and widely regarded as
(Extending) Act, 1934. the “father of the trade union movement”.
11. (b) Sub-Theme: Labour Legislations/Labour He also founded “Mill Hands’ Association”.
Movement If we critically think about statement 1,
Statement 1 is incorrect: The first Factory it states that Factories Act, 1881 allows the
Act (1881) was passed in 1881 during the workers to form trade unions. It was the
tenure of Lord Ripon. This legislation aimed very first Factories Act, 1881. Do you think
to improve working conditions for factory Britishers would give such a heavy right in
workers. This act applies only to factories its very first act rather than incremental
that use mechanical power, employing at moves? This reasoning makes the possibility
least 100 workers. It prohibited employment of statement 1 being incorrect.

52 UPSC Prelims PYQs


Mahalwari System
Read carefully about various personalities,
at least about those which are mentioned
in Modern NCERT. Personalities from Permanent settlement
Swadeshi Movement, Non-Cooperation
Movement, CDM, Quit India Movement (esp. Punjab and
Underground activities) and various socio- UP
religious reform movements are important. Bengal,
12. (d) Sub-Theme: Economic Exploitation of the Bihar and
British/Economic critics of colonialism Orissa
Dadbhai Naoroji, G.Subramania Iyer and R.C. Bombay
Dutt are considered as the economic critics of and
colonialism in India. Madras
Nationalists Critique Of Colonial Economy:
z The early nationalist of India analysed the
colonial character of British rule in India Ryotwari settlement
in great detail. The economic critique
produced by them was one of the remarkable NOTE: Question on land settlement system
achievements of the moderate nationalists. came in 2011 - on land settlement system
z Dadabhai Naoroji (Grand Old man of of Lord Cornwallis.
India): “Poverty and UnBritish Rule in 14. (a) Sub-Theme: Economic Exploitation of the
India” - Propounded Drain of Wealth Theory. British/Economic critics of colonialism
Uncovered the economic exploitation of Statement 1 is correct: Dadabhai Naoroji
India by the British.
(Grand Old man of India) wrote the “Poverty
z Romesh Chandra Dutta: “The Economic and UnBritish Rule in India” a Propounded
History of India” - Half of the annual GDP Drain of Wealth Theory and exposed the
of India outflowed every year. economic exploitation of India by the British.
z G. V. Joshi: Pointed out that the development Statement 2 is incorrect: Swami Dayanand
in India was like a subsidy to the British Saraswati, Swami Vivekananda, etc. interpreted
Industries. the ancient texts and restored the self
z Other Critiques: Justice Mahadev Govind confidence of Indians, not Dadabhai Naoroji.
Ranade, G. Subramaniam Iyer, G.K. Swami Dayananda Saraswati even gave the call
Gokhale etc. examined every facet of the to “go back to vedas”.
economy and subjected the entire range Statement 3 is incorrect: Raja Rammohan Roy,
of economic issues and colonial economic Ishwar Chandra Vidyasagar, Keshav Chandra
policies to minute scrutiny. Sen, Atmaram Pandurang, etc. emphasised the
NOTE: This question can be made correct need for eradication of all social evils before
by reading Modern NCERT. It’s always anything else. Dadabhai Naoroji wasn’t a social
recommended reading between the lines reformer.
about every personality. NOTE: If we carefully look at statement 3,
13. (c) Sub-Theme: Land Revenue Settlement it talks about the “eradication of all the
Statements 1 and 2 are correct: The social evils”. This sounds absolute and
Ryotwari System was started by Thomas exclusive because of the extreme word “all”.
Munro and Alexander Reed. It was started
Eliminating all social evils was a mammoth
in the Baramahal district of Tamil Nadu and
and uphill task. So there is less possibility
later spread to Madras, parts of Bombay, East
Bengal, Assam and Kurg (Karnataka). It of correctness of this statement. If we take
covered 51% of British India. Ryots (farmers) calculable risk of eliminating statement 3,
were given the ownership and other rights we can get an answer. Moreover, Dadabhai
(Pattas) over the land and were required to Naoroji propounded “Drain of wealth
pay the revenue directly to the government. theory”. It was about exposing the economic
Statement 3 is correct: The Ryotwari System
exploitation of India by the British. This
involved high taxes (50% in dryland and 60%
makes statement 1 correct.
in wetland) and strict collection.

Modern History 53
Zamindari System:
Economic Exploitation of the British/
Economic critics of colonialism theme is Zamindari System
significant with respect to Colonial history
in India. In the past, many questions came
from this particular topic to check the
The British Government
analytical ability of aspirants.
15. (d) Sub-Theme: Colonial Trade + Land Revenue
Settlement
Statement 1 is incorrect: After getting the Zamindars
Diwani rights of Bengal, Bihar, and Orissa in
1765 the major aim of the British East India
Company was to increase their land revenue Peasants
collection. So its policies were aimed at getting
maximum income from land without caring of z The Zamindari System was introduced
its consequences on cultivators and peasants. by Lord Cornwallis in 1793 through the
Three major systems of land revenue collection Permanent Settlement Act 1793.
existed in India - Zamindari, Ryotwari and z The land where the zamindars had previously
Mahalwari. been responsible for collecting revenue was
Statement 2 is incorrect: The first recorded given to them as ownership.
use of Artillery in India is at the Battle of Adoni z The settlement is referred to as the
in 1368. In the Deccans the Bahamani kings led Zamindari Settlement since it was made
by Mohammand Shah Bahamani used a train of with the zamindars.
Artillery against the Raja of Vijaya-nagar. z Since they were regarded as the soil’s
tillers, the ryots were made tenants.
Statement 3 is incorrect: Tobacco and Red
z The demand for land revenue was
Chillies were introduced by Portuguese not
predetermined and out of controlled
British. revenue, 10/11th part was given to the
NOTE: In general, assessment of land company.
revenue on the basis of nature of the soil z Company also added the “Sunset clause”
and the quality of crops itself cumbersome for the zamindars.
and administratively mammoth task. It
NOTE: Many questions came from this
also needs a high level of record-keeping
particular theme esp. Permanent Settlement,
practice on part of administration. So this
Ryotwari Settlement, Mahalwari Settlement.
statement sounds. Now, if we carefully go
So understanding differences between them
through Spectrum or Modern NCERT it
stands crucial.
is clear that Portuguese had introduced
Tobacco in India. 17. (b) Sub-Theme: Economic Exploitation of the
British/Economic critics of colonialism
16. (d) Sub-Theme: Land Revenue Settlement
Drain of Wealth and Home Charges:
Reforms of Cornwallis: z Dadabhai Naoroji (Grand Old man of India)
z The Permanent Land Revenue Settlement in his “Poverty and UnBritish Rule in
of Bengal, which was extended to the India” (1901), propounded the Drain of
provinces of Bihar and Orissa, was Lord Wealth Theory.
Cornwallis’ most notable administrative z Home Charges played a significant role
initiative. in the Drain of Wealth from India. It
comprised of:
z The reforms had increased the litigation
 Funding for the Indian offices in London.
after the introduction of the land settlement
 Funds for paying the salaries and
system and the main reason for this was the
pensions of British employees working
removal of Court Fee and now everybody in India.
could drag anybody to the courts.
 Funds used for additional capital
z The extension to the right of appeal was investments, such as railways, and to
also one of the reasons court fees were pay interest on debts.
abolished by Cornwallis and Lawyers were z “Economic history of India” book was
prescribed their fees. written by R. C. Dutta.

54 UPSC Prelims PYQs


Statement 1 is correct: Funds used to support
NOTE: In every alternate year, in one way
the India Office in London. Hence, or another there is a question on acts and
Statement 2 is correct: Funds used to pay policies of colonial times. It is advisable to
salaries and pensions of British personnel go through each and every act/regulation/
engaged in India. law in detail and provision by provision.
Statement 3 is incorrect: No the funds 20. (a) Sub-Theme: Administrative Reform post-1857
collected as Home Charges were never used for Together with Dadabhai Naoroji and other
waging wars outside India by the British. supporters of India, Sir William Wedderburn
and W. S. Caine established the “Indian
18. (c) Sub-Theme: Administrative Reform post-1857 Parliamentary Committee” in 1893. The Indian
Statements 1 and 3 are correct: GoI Act 1919 Parliamentary Committee’s mission is to lobby
introduced the dual scheme of governance the House of Commons for political reforms in
known as ‘dyarchy’ in the executive India.
About Sir William Wedderburn:
government of the provinces. By demarcating
z President of INC: 1838-1918 (Bombay,
and separating the central and provincial 1889, Allahabad, 1910)
subjects, it relaxed the central control over z He came to India in 1860 and registered for
the provinces. It introduced bicameralism and work as an Assistant Collector at Dharwad.
direct elections in the country. The devolution z His interest in these issues led him to contact
of legislative authority by the centre to the the Indian National Congress.
provinces was done in the Government of India z After retiring, Wedderburn threw himself
Act, 1919. into it. He presided over the fourth Congress
held in Bombay in 1889.
Statement 2 is incorrect: The Indian Council
z Together with Dadabhai Naoroji and other
Act, 1909 or the Morley-Minto Reforms
supporters of India, Sir William Wedderburn
introduced separate communal electorates and W. S. Caine established the “Indian
for Muslims. It was put into place to appease Parliamentary Committee” in 1893. The
the moderates (in Congress) and introduces Indian Parliamentary Committee’s mission
separate electorates based on religion. is to lobby the House of Commons for
Therefore, Lord Minto came to be known political reforms in India.
as “Father of the Communal Electorate in z In 1895, Wedderburn represented India
on the Welby Commission (i.e. Royal
India”.
Commission) on Indian Expenditure.
19. (c) Sub-Theme: Administrative Reform post 1857 z He also began participating in the activities
Government of India Act, 1935: of the Indian Famine Union set up in June
z Divided the powers between the Centre 1901, for investigation into famines and
proposing preventive measures.
and provinces in terms of three lists– Federal
z He came to India in 1904 to attend the 20th
List (for Centre, with 59 items), Provincial
session of the Indian National Congress in
List (for provinces, with 54 items) and the Bombay, which was presided over by Sir
Concurrent List (for both, with 36 items). Henry Cotton.
z Abolished dyarchy in the provinces and z He was again invited in 1910 to preside over
introduced ‘provincial autonomy’ in its the 25th session.
place, while adopted the dyarchy at the z He remained the Chairman of the British
Centre Committee of the Congress from July
1889 until his death.
z Introduced bicameralism in six out of eleven
provinces NOTE: By a general understanding of
modern history, we know congress was
z Further extended the principle of communal established in 1885 and 1893 was a
representation to the depressed classes phase of early moderates and demands
(Scheduled Castes), women and labour were mostly concentrated about reforms.
(workers). Demand for India’s Independence came
z Abolished the Council of India, established at a later stage. By this reasoning, we can
eliminate option c.
by the Government of India Act of 1858.

Modern History 55
21. (c) Sub-Theme: Ilbert Bill controversy z According to the proclamation Lord
Ilbert Bill Controversy (1883): Canning became the first viceroy and
governor general of India.
z Two types of law that had been common
in India were what Lord Ripon sought to z The proclamation put an end to the era of
abolish. further expansion of the British Empire
in India.
z The legal system stipulated that a European
could only be tried by a European Judge or Government 0f India Act (1858)
a European Magistrate. z It was known as the “Act of Good
Government of India”.
z The disqualification was unfair, because it
was intended to disgrace and discredit the z According to its provisions, Her Majesty’s
judges who were of Indian descent. government would now be in charge of
India and would do so in her name.
z A bill to end this prejudice in the judiciary
was proposed in 1883 by lawmaker C.P. z It abolished the East India Company and
Ilbert. Europeans were vehemently against transferred powers to the Crown.
this Bill. Statement 1 is correct: The proclamation was
z The Ilbert Bill controversy made educated read out by Lord Canning at a durbar held
Indians aware of the need of an all Indian on November 1, 1858 at Allahabad. It put an
organisation, resulting in the formation of end to the rule of the East India Company and
Indian National Congress in 1885 in Bombay the government of India came directly under
with early leadership of Dadabhai Naroji, the Crown. According to the proclamation
Pherozeshah Mehta, Badruddin Tyabji, WC Lord Canning became the first viceroy and
Banerjee etc. governor general of India. The proclamation
z The Ilbert Bill Controversy is a high put an end to the era of further expansion of
watermark in the history of the Indian the British Empire in India.
National Movement. The immediate result Statement 2 is correct: Queen Victoria’s
of this awakening of India was the birth proclamation of 1858 aimed at ending the rule
of the Indian National Congress in 1885, of the company and transferring all the powers
the very next year of Ripon’s departure. to the British crown after the 1857 revolt.
It also promised non- interference in the
Option (a) is incorrect: The Indian Arms Act
religious affairs of the people, grant of equal
(1878) imposed restrictions and it compelled
protection of law and respect for ancient rights
the Indians to have license to keep, sell or
and customs of the people.
purchase arms.
Statement 3 is incorrect: Queen Victoria’s
Option (b) is incorrect: The Vernacular Press
Proclamation (1858) is aimed at improving the
Act (1878) posed restrictions on newspapers
administrative machinery of supervision and
and magazines published in Indian languages.
control over the Indian government.
Option (c) is correct: The Ilbert Bill
Controversy 1883 was related to the removal 23. (b) Sub-Theme: Administrative Reforms post 1857
of disqualifications imposed on the Indian The Montagu-Chelmsford Reforms which
magistrates with regard to the trial of the became the Government of India Act in 1919,
Europeans. defined the jurisdiction of the central and
Option (d) is incorrect: On March 13th, 1879, provincial governments.
Lord Lytton abolished the import duty on z The GoI Act 1919 introduced a diarchy
cotton goods except on the finest qualities and it (rule of two individuals/parties) for
has no relation with the Ilbert Bill controversy. the executive at the level of the provincial
22. (a) Sub-Theme: Administrative Reforms post 1857/ government.
Queen’s Proclamation z The diarchy was implemented in eight
Queen Victoria’s Proclamation (1858): provinces: Assam, Bengal, Bihar and
z The proclamation was read out by Lord Orissa, Central Provinces, United Provinces,
Canning at a durbar held on November Bombay, Madras and Punjab.
1, 1858 at Allahabad. It put an end to the z The Dyarchy system increased the authority
rule of the East India Company and the of the provincial governments.
government of India came directly under z In the province, the governor was to serve
the Crown. as the chief executive.

56 UPSC Prelims PYQs


Introduction to Diarchy: Introduction to Diarchy:
z The GoI Act 1919 introduced a diarchy (rule z The GoI Act 1919 introduced a diarchy (rule
of two individuals/parties) for the executive of two individuals/parties) for the executive
at the level of the provincial government. at the level of the provincial government.
z The diarchy was implemented in eight z The diarchy was implemented in eight
provinces: Assam, Bengal, Bihar and provinces: Assam, Bengal, Bihar and
Orissa, Central Provinces, United Provinces, Orissa, Central Provinces, United Provinces,
Bombay, Madras and Punjab. Bombay, Madras and Punjab.
z The Dyarchy system increased the authority z The Dyarchy system increased the authority
of the provincial governments. of the provincial governments.
z In the province, the governor was to serve z In the province, the governor was to serve
as the chief executive.
as the chief executive.
Division of Subjects:
Division of Subjects:
z Division of subjects into two lists: ‘reserved’
z Division of subjects into two lists: ‘reserved’
and ‘transferred’.
and ‘transferred’.
z The reserved list: included subjects such
as law and order, finance, land revenue, z The reserved list: included subjects such
irrigation etc. and the subjects were to as law and order, finance, land revenue,
be administered by the governor through irrigation etc. and the subjects were to
his executive council of bureaucrats. All be administered by the governor through
significant subjects were maintained in the his executive council of bureaucrats. All
Provincial Executive’s reserved subjects. significant subjects were maintained in the
z The transferred subjects: were to be Provincial Executive’s reserved subjects.
administered by ministers nominated z The transferred subjects: were to be
from among the elected members of the administered by ministers nominated
legislative council and it included subjects from among the elected members of the
such as education, health, local government, legislative council and it included subjects
industry, agriculture, excise, etc. such as education, health, local government,
24. (d) Sub-Theme: Administrative Reforms/Montague- industry, agriculture, excise, etc.
Chelmsford Reforms Act
NOTE: UPSC has asked several questions
Montague-Chelmsford Proposals had the from this particular theme/sub-theme,
scheme of constitutional reforms, known thus aspirants are requested to read this
as the Montagu-Chelmsford (or Mont-Ford) particular theme very minutely both from
Reforms, which led to the enactment of the Spectrum and Laxmikanth.
Government of India Act of 1919.
z The GoI Act 1919 introduced a diarchy 26. (d) Sub-Theme: Important Commission during
(rule of two individuals/parties) for British
the executive at the level of the provincial Butler Committee (1927):
government. z The Butler Committee was appointed
z The diarchy was implemented in eight under the chairmanship of Sir Harcourt
provinces: Assam, Bengal, Bihar and Butler in 1927 to investigate and clarify
Orissa, Central Provinces, United Provinces, the relationship between the paramount
Bombay, Madras and Punjab. power of the British Raj in India, and the
z The Dyarchy system increased the authority rulers of Princely States.
of the provincial governments. z There were two other members of the
z In the province, the governor was to serve committee: William Searle Holdsworth and
as the chief executive. Sidney Peel.
25. (d) Sub-Theme: Administrative Reforms post 1857/ Recommendations:
Montagu-Chelmsford Reforms/GoI Act 1919 z In the committee’s report of 1929, the
Government of India Act/Montagu- “paramountcy” doctrine was reaffirmed.
Chelmsford Act, 1919 divided the provincial z Guidelines were given for its application,
subjects into two parts: transferred and and it was made clear that the financial
reserved, this dual scheme of governance was relationship between the Raj and the States
known as ‘dyarchy’. should be fair.

Modern History 57
z The relationship of the princely states z Taxes could be levied against citizens by the
with the British empire was not merely a local government. They could punish people
contractual relationship, but a living, growing for not paying taxes.
relationship shaped by the circumstances z It ended the East India Company’s monopoly
and policy, resting on the mixture of history on trading with India.
and theory.
z The company’s rule was extended for
z British paramountcy to stay intact(solid) to
another 20 years. The trade monopoly of
preserve the princely state.
their country was ended except for the trade
z State should not be transferred without their in tea, opium, and with China.
own consent to a relationship with a new
government in British India responsible to z Giving missionaries permission to travel to
an Indian legislature. India was one of the key components of this
law.
27. (b) Sub-Theme: Administrative Reforms post 1857
PITT’S INDIA ACT OF 1784:
As per the Government of India Act of
z Pitt’s India Act, 1784 extended the control
1935 Governor-General was empowered
of the British Government over the
to authorise, either the Federal or the
Provincial Legislature to enact a law with Company’s affairs and its administration
respect to any residuary matter. in India.
The Government of India Act, 1935 The act was significant for two reasons:
z Divided the powers between the Centre and z The Company’s possessions in India were
provinces in terms of three lists– Federal officially referred to as “British possessions
List (for Centre, with 59 items), Provincial in India” for the first time.
List (for provinces, with 54 items) and the z The Company’s activities and administration
Concurrent List (for both, with 36 items). in India were placed under the total control
z The allocation of residuary powers was of the British Government.
unique. It was not vested in either of the Statement 1 is correct: The Charter Act 1813
legislatures, central or provincial. But
ended the trade monopoly of the East India
the Governor-General was empowered
Company except for the trade in tea, opium,
to authorise, either the Federal or the
and with China.
Provincial Legislature to enact a law with
Statement 2 is correct: Company’s territories
respect to any residuary matter.
in India came under the Sovereignty of the
z Abolished dyarchy in the provinces and
British Government through this Act.
introduced ‘provincial autonomy’ in its
Statement 3 is incorrect: The revenue of
place, while adopted the dyarchy at the
Centre India was controlled by the British parliament
through Pitt’s India Act 1784.
z Enacted bicameralism in six out of eleven
provinces 29. (b) Sub-Theme: Administrative Reforms post 1857
z It further extended the principle of Montagu-Chelmsford Act, 1919:
communal representation to the depressed z by demarcating and separating the central
classes (Scheduled Castes), women and and provincial subjects, it relaxed the central
labour (workers). control over the provinces
z Abolished the Council of India, established z divided the provincial subjects into two parts
by the Government of India Act of 1858. – transferred and reserved, this dual scheme
of governance was known as ‘dyarchy’
NOTE: Questions from this particular theme
were generally straightforward in nature, z introduced bicameralism and direct
also this particular theme is a very high elections in the country.
yielding theme both in terms of Modern z It required 3 out of 6 members in the
History and Polity, so read thoroughly as Viceroy’s Executive Council to be Indian.
you cannot afford to miss these low hanging z recommended the voting rights to women
fruits. in limited numbers to be extended on the
28. (a) Sub-Theme: Administrative Reforms before 1857 basis of property, tax or education
CHARTER ACT OF 1813: z extended the principle of communal
z Company’s territories in India came under representation among the Sikhs, Indian
the Sovereignty of the British Government. Christians, Anglo-Indians and Europeans.

58 UPSC Prelims PYQs


z establishment of a public service z There were restrictions imposed on the
commission. utilisation of forest products, shifting
z separated the provincial budgets from the agriculture, and hunting methods. This
Central budget led to the loss of livelihood for the tribals.
Government of India Act, 1935: z The tribals turned into landless agricultural
z divided the powers between the Centre and labourers.
provinces in terms of three lists– Federal z Money lenders were introduced by the
List (for Centre, with 59 items), Provincial British into the tribal regions, which greatly
List (for provinces, with 54 items) and the increased the exploitation of the native
Concurrent List (for both, with 36 items). tribes. Under the new economic structure,
z abolished dyarchy in the provinces and they were reduced to bonded labour.
introduced ‘provincial autonomy’ in its z A joint ownership arrangement for land
place, while adopted the dyarchy at the existed in tribal communities before the
Centre idea of private property took hold.
z introduced bicameralism in six out of eleven z Tribal society has historically been
provinces egalitarian in contrast to mainstream
society, which is characterised by caste and
z further extended the principle of
class inequalities. However, with the arrival
communal representation to the
of non-tribals or outsiders, tribal people
depressed classes (Scheduled Castes),
began to be seen as belonging to the lowest
women and labour (workers).
strata of society.
z abolished the Council of India, established
z In order to manage the abundant riches of
by the Government of India Act of 1858. Indian forests, the government established a
Statement 1 is incorrect: Although the Forest Department in 1864. A comprehensive
Montague Chelmsford Act or the Govt of India government monopoly on the forested land
Act, 1919 recommended the voting rights to was created by the Govt Forest Act of 1865
women in limited numbers to be extended on and the Indian Forest Act of 1878.
the basis of property, tax or education but it z The work of Christian missionaries also led
did not recommend granting voting rights to to social unrest in tribal societies, which
all women above the age of 21. also aroused their resentment.
Statement 2 is correct: The Government of
z Major Tribal movements of 19th Century:
India Act 1935 does reserve seats for women Kol Rebellion (1832); Santhal Rebellion
in the legislature, it extended the communal (1855-56); Munda Rebellion (1899-1900)
representation by giving a separate electorate
to women. NOTE: option (d) sounds more inclusive
and wider in coverage in terms of issues
NOTE: This is a very repetitive and high
faced by tribals. It also includes the meaning
yielding theme/sub-theme. Go through
and spirit of the above 3 sentences i.e. a, b,
the series of British policies and acts very
and c in one way or other. The options are
carefully.
ambiguous, hence we stuck to the official
30. (d) Sub-Theme: Tribal Movement answer key of UPSC.
The essence of Tribal life, i.e. shifting 31. (a) Sub-Theme: Peasant Movement
cultivation, hunting, fishing and the use of Tebhaga Peasant Movement (1946) in
forest produce etc. was completely disrupted Bengal demanded the share of the landlords
during the colonial rule. to be reduced from one-half of the produce
Causes of Tribal Revolts in India in the 19th to one third of the produce.
Century: Tebhaga Movement (1946):
z The cornerstone of Tribal life was shifting z In September 1946, the Bengal Provincial
cultivation, hunting, fishing and the use Kisan Sabha gave a call to implement,
of forest produce. through mass struggle, the Floud
z The practice of settled agriculture was Commission recommendations of
established with the inflow of non-tribals Tebhaga which means two-thirds’ share
into the traditional tribal lands and as a to the bargardars, the share-croppers also
result of it the tribal population suffered a known as ‘bhagchasi’ or ‘adhyar’, instead of
land loss. the one-half share.

Modern History 59
z The jotedars rented the land that the enacted laws so that the lands of tribals
bargardars worked on. could not be taken by outsiders (dikus).
z The main slogan was “nij khamare dhan 33. (b) Sub-Theme: Peasant Movement
tolo,” which meant that sharecroppers
z Indigo cultivation in India declined by the
should thresh their own paddy and not
beginning of the 20th century because of
to the jotedar’s house, as before, so as to
enforce tebhaga. new inventions of improved versions of
chemical colours that substitute indigo,
z North Bengal was the epicentre of the
thus the production of indigo became
movement, particularly among the
Rajbanshis, a low caste with tribal origins. unprofitable in the world market. Thus, its
Muslims took part in great numbers as well. unprofitability in the world market because
of new inventions’ caused decline in the
z The movement dissipated soon, as the
League ministry’s sop of the Bargardari Bill, Indigo Cultivation.
the popularisation of the Hindu Mahasabha’s Indigo Revolt 1856-57:
campaign for a separate Bengal, and new z Area: Bengal
riots in Calcutta, which eliminated the z Leaders: Bishnu Charan Biswas and
possibility of sympathetic support from the Diagambar Biswas
urban sections. z The great worth of indigo as a commerce
NOTE: If we read or Modern NCERT commodity led to its frequent usage as the
carefully, Tebhaga literally means ‘three nickname “blue gold.” It gives the materials
shares’ of harvests. Option 1 talks about a blue tint.
the “one-third” part. With just knowing z Reason: They started losing their land due
the meaning of “Tebhaga’’, we can solve the to poor output and they started fall into debt
given question. trap
32. (c) Sub-Theme: Tribal Movement z India’s first peasant strike resulted in the
Both the given measures were taken by the compelled planting of indigo since the trade
Colonial Government as a result of Santhal in it was profitable because blue dye was in
Uprising. high demand.
Santhal Uprising:
z During their strike, peasants refused to
z Santhals: They used to live in the Daman-
produce indigo or sign contracts. The strike
i-Koh area which is between Bhagalpur and
Rajmahal, and ‘hool’ means uprising in the expanded to other parts of Bengal, and
Santhali language. other zamindars supported the uprisings. A
commission was established in their favour.
z The Santhals of the Rajmahal Hills were
resentful of the oppression they suffered at z Intelligentsia helped to make it a powerful
the hands of revenue officials, police, money- campaign.
lenders, and landlords, by the “outsiders” z Harsh Chandra Mukherjee (editor Hindu
(whom they called diku). patriot), Dinabandhu Mitra play: Neel
z In 1854, the Santhals under the leadership Darpan.
of Sido and Kanhu rebelled against their 34. (d) Sub-Theme: Tribal Movement
oppressors, declared the end of the
Company’s control, and proclaimed their Munda Rebellion:
independence. z It is one of the prominent 19th century tribal
rebellions in the subcontinent. In the years
z The situation wasn’t brought under control
until 1856, a year that saw extensive military 1899–1900, Birsa Munda served as the
actions. Sido died in 1855, while Kanhu movement’s leader in the south of Ranchi.
was arrested in 1866. z The Ulgulan, also known as “the Great
z A separate district of Santhal Parganas was Tumult,” was an insurrection that aimed
created by the Government to pacify the to establish Munda Raj or Munda authority
Santhals. Thus, the government separated in this area.
Santhal Parganas into its own district in Reason of revolt:
order to pacify the Santhals. z In 1874, the British replaced the traditional
z It became illegal for a Santhal to transfer Khuntkari system of Munda tribals with
land to a non-Santhal. British Government the zamindari system.

60 UPSC Prelims PYQs


z The introduction of the zamindari system z Later Debendranath Tagore joined the
created the classes of zamindars (landlords) Brahmo Samaj in 1842.
and ryots (tenants). z Keshab Chandra Sen was appointed as
z This led to the monetisation of the Acharya of the Brahmo Samaj in 1858.
economy, the tribals had to depend on cash z Favoured: Human reason and conscience;
for paying the rent and for meeting their Political upliftment of the masses, widow
daily needs. remarriage.
z This situation left them with no other option z Against: Polytheism; Incarnations;
but to depend on the money-lenders, who Scriptural authority; Idol worship; Caste
used to lend money at an exorbitant rate. system; Untouchability; Sati; Child marriage;
Birsa Munda: Purdah system.
z A tribal liberation warrior from the Munda z No definite view: on Karma and the
tribe, Bisra Munda is revered as a folk hero. transmigration of the soul.
z He was a spearhead behind the Ulgulan z Punjab: Dyal Singh College at Lahore (1910)
movement that arose in the Chotanagpur started by Dayal Singh Trust, popularised
region of Bihar and Jharkhand belt in the the ideas of the Samaj.
19th century under the British colonisation. Statement 1 is correct: Raja Ram Mohan Roy
z He is often referred to as the ‘Dharti Abba’ founded the Brahmo Samaj (1828) [formerly
or the Earth Father. called Brahmo Sabha.] Later Debendranath
Tagore joined the Brahmo Samaj in 1842.
z Birsa Munda organised masses to stop Brahmo Samaj vehemently opposed idolatry or
paying debts/interest to moneylenders and idol worship.
taxes to the British. By this, he led a revolt Statement 2 is correct: The chief aim of
to bring an end to Victorian rule and the Brahmo Samaj was the worship of the eternal
establishment of Munda Rule in Jharkhand God. It was against priesthood, rituals and
(Chotanagpur region). sacrifices.
NOTE: If we go through Modern Ncert in Statement 3 is incorrect: Brahmo Samaj
detail, this question can be easily solved. It spread the religious treasure and teachings
is advisable to remember keywords from of Vedas but never claimed that Vedas are
Ncert such as Dikus, Ulgulan, Tebhaga, nij infallible.
khamare dhan tolo etc. and corresponding 37. (c) Sub-Theme: Social Reform Organization
movements. An anti-caste movement was organized by
35. (b) Sub-Theme: Social Reform Organizations Satya Shodhak Samaj in Maharashtra.
Satyashodhak Samaj:
Indian Social Conference (1887):
z On 24 September 1873, Jotirao Govindrao
z It was formed by M. G. Ranade and
Phule formed Satyashodhak Samaj to
Raghunath Rao in Madras. focus on the rights of depressed groups such
z It is a separate social reform organ of as women, the Shudra, and the Dalit.
the Indian National Congress as they did z The Satya Shodhak Samaj opposed idolatry
not want to include social reforms in their and condemned the caste system by doing
deliberations. this.
z Advocated inter-caste marriage, opposed z The Satyashodhak Samaj opposed the need
polygamy and kulinism. for priests and advocated for the spread
z Launched “Pledge Movement to pledge of reason.
against Child Marriage” z Jotirao Govindrao Phule founded
36. (b) Sub-Theme: Social Reform Organisations/ Social Satyashodhak Samaj with the goals of
Reformist promoting the happiness, well-being,
equality, and simplicity of religious practices
Brahmo Samaj did oppose idolatry and
for all people.
denied the need for a priestly class for
interpreting the religious texts. z The Deenbandhu newspaper, published
BRAHMO SAMAJ (1828) from Pune, gave the Samaj’s viewpoints a
z Raja Ram Mohan Roy founded the Brahmo voice.
Samaj (1828) [formerly called Brahmo z Muslims, Brahmins, and government officials
Sabha.] all belonged to the samaj. Leading members

Modern History 61
and financial backers for the group came z The case was instrumental in the
from Phule’s own Mali caste. development of the Age of Consent Act in
1891.
38. (b) Sub-Theme: Social Reform Movement/Social
Reformist Statement 1 is incorrect: Dr. Rakhmabai
was an Indian physician and a feminist. At
Keshab Chandra Sen (1838-1884):
the young age of eleven, she was married to
z Appointed as Acharya of the Brahmo
Dadaji Bhikaji, who was then nineteen. Bhikaji
Samaj in 1858. petitioned the court in 1885, 12 years after they
z Spread the ideas of the Samaj outside had been married, asking for the “restitution
Bengal (in United Provinces, Bombay, of conjugal rights.”The Rakhmabai case did
Punjab, etc.). not involve the women’s right to gain education.
z Showed radical views against caste Statement 2 is correct: The Rakhmabai case
system, favored inter-caste marriage. of 1884 was instrumental in the drafting of the
Age of Consent Act in 1891. Hence, statement
z Expelled from Acharya’s position in 1865.
2 is correct.
z Founded Brahmo Samaj of India (1866). Statement 3 is correct: Dr. Rakhmabai was
z Ananda Mohan Bose, Shibchandra Deb, married off at the young age of eleven with
and Umesh Chandra Datta created the Dadaji Bhikaji, who was nineteen years old. In
Sadharan Brahmo Samaj in opposition to 1885, after 12 years of marriage, Bhikaji filed a
his ideas. petition in the court and sought “restitution of
z Keshab Chandra Sen erected the mandir conjugal rights”. Hence, statement 3 is correct.
known as the Tabernacle of New 40. (d) Sub-Theme: Revolutionary Activities Abroad
Dispensation on January 24, 1868, in Ghadar Party, 1913:
honour of the Magha Festival.. z Ghadar Party was a revolutionary group
z On October 29, 1870, the Indian Reform organised around a weekly newspaper
Association was founded with Keshab “The Ghadar” with its headquarters at San
Chandra Sen as President. It comprised Francisco and branches along the US coast
many people who did not belong to the and in the Far East.
Brahmo Samaj and represented the secular z These revolutionaries included mainly
wing of the organisation. ex-soldiers and peasants who had migrated
from the Punjab to the USA and Canada in
z The objective was to put into effect some
search of better employment opportunities.
of the concepts Sen was exposed to while
z Pre-Ghadar revolutionary activity had been
in Britain.
carried on by Ramdas Puri, G.D. Kumar,
39. (b) Sub-Theme: Social Reformist/Social Reform Taraknath Das, Sohan Singh Bhakna and
Organization Lala Hardayal who reached there in 1911.
Dr. Rakhmabai (1864–1955) z To carry out revolutionary activities, the
z Dr. Rakhmabai was an Indian physician earlier activists had set up a ‘Swadesh
and a feminist. Sevak Home’ at Vancouver and ‘United
z Apart from being one of the first practising India House’ at Seattle. Finally, in 1913,
women doctors in colonial India, she the Ghadar was established, with the aim
was also involved in a landmark legal case of organising assassinations of officials,
involving her marriage as a child bride publishing revolutionary and anti-
imperialist literature, work among Indian
between 1884 and 1888.
troops stationed abroad, procure arms and
z At the young age of eleven, she was married bring about a simultaneous revolt in all
to Dadaji Bhikaji, who was then nineteen. British colonies.
Bhikaji petitioned the court in 1885, z The moving spirits behind the Ghadar
12 years after they had been married, asking Party were Lala Hardayal, Ramchandra,
for the “restitution of conjugal rights.” Bhagwan Singh, Kartar Singh Saraba,
z Behramji Malabari and Pandita Ramabai Barkatullah and Bhai Parmanand.
established the Rakhmabai Defense Other prominent leaders involved in the
Committee to stand up for her. movement: Kartar Singh Saraba, Raghubar
z The case lasted for 4 years until Dadji was Dayal Gupta, Rash Behari Bose, and Sachin
compensated in 1888 out of court. Sanyal.

62 UPSC Prelims PYQs


Additional Information: 43. (a) Sub-Theme: Revolutionary Activities Abroad
Barindra Kumar Ghosh was associated with About the Ghadar:
a revolutionary group called Jugantar and z The Ghadar Party was a revolutionary
Jogesh Chatterjee was associated with the organisation centred on the weekly
Hindustan Republican Association/Army newspaper “The Ghadar,” which had
(HRA). branches in the Far East and along the
41. (a) Sub-Theme: Phase of Mahatma Gandhi US coast in addition to its San Francisco
Statement 1 is correct: Kheda Satyagraha headquarters.
(1918) was the first Non-cooperation z Ramdas Puri, G.D. Kumar, Taraknath Das,
Movement. Crops failed in Kheda district of Sohan Singh Bhakna, and Lala Hardayal,
Gujarat because of drought. The yield was less
who arrived there in 1911, were involved in
than 1/4th of normal produce so farmers were
pre-Ghadar revolutionary activities.
entitled to remission according to revenue
code but the Government demanded taxes z The Ghadar programme aimed to organise
and ordered seizure of property, if not paid. official assassinations, disseminate anti-
Mahatma Gandhi asked the farmers not to pay imperialist and revolutionary literature,
the taxes and organise a satyagraha on behalf coordinate with Indian troops stationed
of the peasants. Leaders: Sardar Vallabhai abroad, get weapons, and spark a
Patel, Mohanlal Pandya, Narhari Parekh, simultaneous uprising across all British
etc. Government finally agreed to return all colonies
confiscated property, reduce the increase in z The Ghadar Party’s inspirational forces were
tax, and suspend the tax for the year. Lala Hardayal, Ramchandra, Bhagwan
Statement 2 is incorrect: Permanent Singh, Kartar Singh Saraba, Barkatullah,
Settlement was introduced by Lord and Bhai Parmanand.
Cornwallis in 1893 and it was confined in
z The Ghadarites intended to bring about a
the areas of Bengal, Bihar, Orissa, Varanasi
revolt in India. Their plans were encouraged
(United Provinces), Northern Karnataka. It
by two events in 1914, the Komagata
was never introduced in Gujarat.
Maru incident and the outbreak of the
42. (b) Sub-Theme: Partition of Bengal
First World War.
Partition of Bengal (1905):
z The decision to divide Bengal was made 44. (a) Sub-Theme: Swadeshi Movement
public by the British Government in 1903 Swadeshi and Boycott Movements:
and partition came into force in October z It was launched due to the partition of
1905. Bengal. Later, the movement spread to other
z Reasons given by British- Bengal are too regions of the nation:
big to be administered and this division  Poona and Bombay: Bal Gangadhar
of Bengal into eastern and western Bengal Tilak
would develop Assam.  Punjab: Lala Lajpat Rai, Ajit Singh
z Moderates started anti partition campaigns:  Delhi: Syed Haider Raza
z Leaders: Surendranath Banerjee, K.K Mitra  Madras: Chidambaram Pillai
and Prithwishchandra Ray
z The Swadeshi Movement contributed to
z Newspaper: Hitabadi, Sanjibani, Bengalee.
a split in the Indian National Congress
z The Partition of Bengal was annulled during in 1907, resulting in the emergence of
the visit of the Royal Majesties, George V and ‘moderates’ and ‘extremists’.
Queen Mary to India. Partition of Bengal
z Congress Session 1907, Surat, presided
was annulled mainly to curb the rising
revolutionary extremism/terrorism. by Rash Behari Ghosh was known as Surat
This action disappointed the Muslim Split.
political elite. King George V announced the z Extremists wanted either B.G. Tilak or
abrogation in the Delhi Durbar on 11th Lajpat Rai as President.
December, 1911. King George V abrogated z Moderates proposed Rashbehari Ghosh
Curzon’s Act as the Royal Durbar in Delhi and wanted the session to be in Surat in
in 1911. order to exclude Tilak from the presidency.

Modern History 63
z Since a leader from the host province could 47. (c) Sub-Theme: Phase of Mahatma Gandhi
not be session president, they also sought to Champaran Satyagraha (1917):
drop resolutions on swadeshi, boycott and z Mahatma Gandhi’s initial experiment of
national education. satyagraha was known as Champaran
45. (b) Sub-Theme: Important INC Sessions/Surat Split/ Satyagraha.
Swadeshi Movement z It was undertaken after Mahatma Gandhi
Option (a) is incorrect: Morley-Minto learned about the abuses suffered by
Reforms, 1909 introduced separate farmers, who were forced into growing
electorates on the basis of religion, indigo by British planters and estate owners.
thus introducing a system of communal z The tenants from Champaran were forced
representation. This was not the reason for the under the law to grow indigo on 3/20th
Congress split in 1907. part of the land (Tinkathia System) for
Option (b) is correct: On the backdrop of the his landlord.
Swadeshi Movement, the Extremists wanted
z To increase their profits, European planters
either B.G. Tilak or Lajpat Rai as President.
demanded exorbitant rents and illegitimate
Moderates proposed Rashbehari Ghosh and
fees.
wanted the session to be in Surat in order
to exclude Tilak from the presidency. Since z Rajkumar Shukla invited Gandhi to look
a leader from the host province could not be into the problems of Indigo planters in
session president, they also sought to drop Champaran, Bihar.
resolutions on swadeshi, boycott and national z Gandhiji intended to conduct a thorough
education. Thus it resulted in a split in the investigation in the district and demand
congress. Therefore, Extremists’ lack of faith in action based on its results.
the capacity of the moderates to negotiate with z During this time Gandhiji was even charged
the British Government’. with violating law and was told to leave
Option (c) is incorrect: Formation of Muslim Champaran, but he refused to leave.
League in 1906 at Dhaka was not the reason
z Later, the then Lieutenant Governor of Bihar
for Congress split in 1907.
ordered the withdrawal of the case against
Option (d) is incorrect: This is irrelevant to
Gandhi, and the Collector wrote to Gandhi
the particular question.
saying he was free to conduct the inquiry.
NOTE: Question on similar theme/topic i.e z In the history of the independence struggle,
Surat Split/Swadeshi Movement came in
this tiny step in the form of passive protest
2015.
was a great leap, signalling the beginning of
46. (a) Sub-Theme: Swadeshi Movement the Gandhian period.
Option (a) is correct: The ‘Swadeshi’ and z Leaders associated: Rajendra Prasad,
‘Boycott’ were the mass movements to oppose Mazhar-ul-Haq, Mahadeo Desai, Narhari
the Bengal partition. Parekh, J.B. Kripalani etc.
Option (b) is incorrect: The Home Rule
z Gandhi’s position in India’s resistance to the
Movement in 1916 was the Indian response
British raj was solidified by the victory at
to the First World War in a less charged but
Champaran.
in more effective way. With people already
feeling the burden of war time miseries caused NOTE: in given statements, notice “all-
by high taxation and a rise in prices, Tilak and India participation” in statement (a)
Annie Besant ready to assume the leadership, and “Drastic decrease” in statement
the movement started with great vigour. (d), we can take risk of eliminating these
Option (c) is incorrect: The time period of extreme statements. Always remember,
Swadeshi and Boycott Movement and Non- there is no set rule for this “extreme word”
Cooperation Movement is different. Therefore elimination method. There are also some
it has no correlation with the Swadeshi and instances where sentences with extreme
Boycott Movement. words were correct, for instance a question
Option (d) is incorrect: Simon Commission
in 2017 about Foreign exchange of India. In
visited in 1928 to review the constitutional
the mentioned question, the statement with
reform in Britain’s largest and most important
“drastic increase” was correct.
possession.

64 UPSC Prelims PYQs


48. (c) Sub-Theme: Swadeshi Movement z Recommendations:
Swadeshi and Boycott Movements:  Dominion status on lines of self-
z There were large-scale protests against the governing dominions. Therefore,
partition of Bengal. Later, the movement statement 1 is incorrect.
spread to other regions of the nation:  Rejection of separate electorates. Joint
z The Boycott Resolution was passed at the electorates with reservation of seats for
Calcutta Town Hall on August 7, 1905, Muslims at the Centre and in provinces
formally launching the Swadeshi Movement. where they were in minority. Therefore,
z Partition came into force: October 16, 1905 statement 2 is correct.
z There were also new means of protest  Linguistic provinces.
like processions, public meetings, and  Nineteen fundamental rights,
boycotts of imported goods. including the rights to form unions, the
z Imaginative use of Traditional Festivals, right to equal treatment for women,
Melas: Tilak’s Ganapati and Shivaji festivals and universal adult suffrage. Therefore,
became a medium of swadeshi propaganda. statement 3 is correct.
Statement 1 is correct: Importance to Self-  Responsible government at the Centre
Reliance (Atma Shakti): Emphasis was placed and in provinces.
on honour, social and economic regeneration of  Full protection of the cultural and
the villages. Swadeshi enterprises: Swadeshi religious interests of Muslims.
textile mills, banks, etc, were set up. At  Complete dissociation of State from
Tuticorin, V.O. Chidambaram Pillai founded the religion.
Swadeshi Steam Navigation Company.  The three Indian Round Table Conference
Statement 2 is correct: Swadeshi programme (1930–1932) attendees had access to
or National Education:
both the Nehru Report and the Simon
1. Bengal National College - Aurobindo
Commission report.
Ghosh as its Principal - inspired by Tagore’s
 Nehru and Subash Bose rejected
Shantiniketan.
the congress goal and set up the
2. The National Council of Education (1906)
Independence for India league.
was set up to organise national education in
the vernacular medium. NOTE: If we read NCERT of Modern India,
3. The Bengal Institute of Technology was we will come to know “Dominion Status”
established to provide technical training. for India was the principal demand of the
z Cultural Impact: Rabindranath Tagore Nehru report. Just knowing this basic fact,
wrote Amar Sonar Bangla, Subramania we can eliminate option 1 and you will get
Bharati wrote Swadesha Geetham. the answer.
z Corps of Volunteers or ‘Samitis’: they 50. (b) Sub-Theme: Phase of Mahatma Gandhi/Acts and
generated political consciousness among Regulation under the Crown Rule
the masses. Option (a) is incorrect: The drain on the
Indian economy in the form of cash, kind and
NOTE: Question on similar theme/topic i.e loans to the British government came to about
Surat Split/Partition of Bengal/Swadeshi 367 million pounds during the WW-I. Rowlatt
Movement came in 2014 (on annulment of Act has no relation with draining Indian money
partition of Bengal), 2015 and 2016 (total or economic support to war efforts.
2 questions). Hereby, revising PYQ stands Option (b) is correct: Anarchical and
important. Revolutionary Crimes Act/ Rowlatt act gives
49. (b) Sub-Theme: Nehru Report the notorious power to imprison activists
Nehru Report (1928): without trial for two years, even possession
z As an answer to Lord Birkenhead’s of seditious newspapers is adequate evidence
challenge, the Nehru report was prepared of guilt.
by a committee headed by Motilal Nehru, Option (c) is incorrect: The Khilafat Movement
the committee included Tej Bahadur Sapru, (1919-20) was a movement to express Muslim
Subhash Bose, M.S. Aney, Mangal Singh, Ali support for the Caliph of Turkey, to protect the
Imam, Shuab Qureshi and G.R. Pradhan as institution of the Khalifa in Turkey, against the
its members. allied powers, particularly Britain. The Rowlatt

Modern History 65
Act was not aimed at suppressing the Khilafat to abstain from the First Round Table
Movement. Conference because the British Government
Option (d) is incorrect: Vernacular Press declined to call a Constituent Assembly to
Act, 1878, The Newspaper Act, 1908 and The draft a constitution for India. Launch of a
Indian Press Act, 1931 were passed in order civil disobedience movement for complete
to impose restrictions on freedom of press. It independence and 26 January to be observed
has no relation with the Rowlatt Act. as ‘Independence Day’/Swarajya Day were
51. (a) Sub-Theme: Important Sessions of INC also decided in the Lahore Session.
Statement 1 is correct: In the Lahore Session Option (c) is incorrect: Mahatma Gandhi
of INC(1929), the resolution on ‘Poorna moved the Non-cooperation resolution in
Swaraj’ or complete independence was the Special Calcutta Session of Congress in
passed. 1920.
Statement 2 is incorrect: The rift between Option (d) is incorrect: Decision to participate
the extremists and moderates took place in the in the Round Table Conference in London was
Surat Session(1907) and the split was resolved taken in the Karachi Session of Congress in
in the Lucknow Session 1916. 1931.
Statement 3 is incorrect: The Cabinet
NOTE: Question on the Lahore Session of
Mission recommended an undivided India and
the Indian National Congress (1929) earlier
rejected the demand of the Muslim League for
came in 2012. This particular theme is a
a separate Pakistan. The Cabinet Mission felt
high-yielding theme and if the aspirants
that a separate state would contain a larger
cover this theme/sub-theme diligently then
proportion of the Non-Muslim population and
questions from this section may appear
a sizable population of Muslims will be left in
as low-hanging fruits which an aspirant
India, if two states are declared.
cannot miss.
52. (c) S u b -T h e m e : A d m i n i s t r a t i v e R e f o r m /
54. (b) Sub-Theme: Phase of Mahatma Gandhi
Constitutional Reforms
Statement 1 is correct: Rowlatt Act was
Simon Commission (1927):
z The Indian Statutory Commission, passed on the recommendations of the Sedition
commonly referred to as the Simon Committee chaired by Sir Sidney Rowlatt.
Commission, was a group of seven Statement 2 is correct: In organising his
British Members of Parliament under the satyagraha, Gandhiji tried to utilise three types
chairmanship of Sir John Simon who of political networks: The Home Rule Leagues;
arrived in British India in 1928. Certain Pan-Islamist groups and Satyagraha
z Objective: The Simon Commission was Sabha.
appointed by the British government to Statement 3 is incorrect: The two time
assess India’s constitutional development periods are not the same. It has a gap of 9 years,
and recommend constitutional reforms. the incident of Rowlatt Satyagraha took place
z Many Indians strongly opposed the in 1919 and the Demonstrations against the
Commission for a variety of reasons. The arrival of Simon Commission took place in
main reason behind the agitation against 1928, thus the two events cannot coincide.
Simon Commission was that there were NOTE: Question on Rowlatt act came
no Indian members of the panel and in 2012 as well. Moreover, the Rowlatt
only seven British members of the British Satyagraha took place in 1919 and the Simon
Parliament. This was viewed as racist and Commission in 1928. So the possibility of
colonialist. coincidence of Simon Commission with
53. (b) Sub-Theme: Important Sessions of INC Rowlatt Satyagraha stands null. Point
to note here is that the relative idea of
Option (a) is incorrect: Attainment of Self-
timeline and corresponding events is
Government was declared as the objective of
very important. This helps us to solve or
the Congress in the Calcutta Session, 1906.
eliminate statements.
Option (b) is correct: Attainment of Poorna
Swaraj was adopted as the goal of the 55. (a) Sub-Theme: Home Rule League Movement
Congress in the Lahore Session, 1929 under Home Rule League Movement (1916):
the presidentship of Jawaharlal Nehru. Along z It was an Indian response to World War - I,
with this, the Congress made the decision inspired by Irish Home Rule Leagues.

66 UPSC Prelims PYQs


z The movement sought to achieve self-rule 2. New Constitution making Body and the
through political discussion and education, Government would negotiate a treaty
public gatherings, etc. for Transfer of Power.
z The government responded with severe 57. (b) Sub-Theme: Quit India Movement
repression, but it prepared masses for Quit India Movement/August Revolution
Gandhian style of politics. (1942):
z Tilak launched the Indian Home Rule z After failure of Cripps’s Mission, a
League in April 1916 at Belgaum. Resolution was framed by Gandhiji for
z Annie Besant launched the Home Rule British withdrawal and non-violent Non-
League in September 1916 at Madras. Cooperation Movement against Japanese
z The All India Home Rule League became Invasion.
Swarajya Sabha in 1920. z Resolution was accepted in the CWC meeting
in Wardha on July 14, 1942.
z Its demands included Swarajya, formation
of linguistic states and education in the z AICC Gowalia Tank Meeting, Bombay:
vernacular.  The Quit India Resolution was ratified in

z Leaders: Motilal Nehru, Lala Lajpat a Congress meeting at Gowalia Tank on


Rai, Jawaharlal Nehru, Bhulabhai 8th August 1942.
Desai, Chittaranjan Das, Madan Mohan  Apart from QIM, other resolutions

Malaviya, Mohammad Ali Jinnah, Tej involved the Civil Disobedience


Bahadur Sapru. Movement against British Rule.
 Mantra of “Do or Die” was adopted in
z Anglo-Indians, most of the Muslims and Non
brahmins from south did not join as they this Session by Gandhiji.
felt it would mean home rule of the Hindu z Major activity after QIM resolution was
majority. passed:
 It was started in response to Mahatma
56. (b) Sub-Theme: Cripps Mission
Gandhi’s national appeal for satyagraha.
Cripps Mission (1942):
Gandhiji advocated for “an orderly
z In March 1942, a mission headed by Stafford British withdrawal” from India as
Cripps was sent to India with constitutional the All-India Congress Committee
proposals to seek Indian support for World announced a nationwide protest.
War II. Because of the reverses suffered
 Destruction of symbols of Public
by Britain in South-East Asia, the Japanese
Authority.
threat to invade India seemed real now and
 Underground activities: Usha Sharma
Indian support became crucial.
started an underground Radio to give
z There was pressure on Britain from the
fuel to the movement.
Allies (USA, USSR, and China) to seek Indian
 Yusuf Meherally, a communist and
cooperation.
trade unionist who also held the office
Provisions: of Mayor of Mumbai, is credited with
z Indian union with a Dominion status coining the phrase “Quit India.”
would be set up and it would be free to
 Parallel Government: established in
decide its relation with the UN and the
Ballia (UP), Tamluk (Bengal) organised
Commonwealth.
“Bidyut Bahini”, Satara (Maharashtra)
z After the war the Constituent Assembly “Prati Sarkar” was organised by Y. B.
would be formed with members from Chavan, Nana Patil etc.
Provinces (elected through proportional  Youth, Women, Workers, Peasants,
representation) and Princely states Government officials especially of
(nominated). (Hence, statement 1 is lower levels, Muslims, Communists
incorrect.) participated in the movement. Upper
z Conditions of Accepting New Constitution: class remained Loyalists.
1. Any Province not willing to join the  There were no communal clashes during
Union can have a separate Constitution the movement. But there was severe
and form a Separate union. (Hence, government repression with upto
statement 2 is correct.) 10,000 being killed.

Modern History 67
Option (a) is correct: The movement basically were able to utilise the council work to their
promoted the non-violent and non-cooperative advantage to a great extent and ruled for 28
movement. months.
Option (b) is incorrect: The British Govt Resignation of Congress Ministries (1939):
responded to the call of Gandhi by arresting z The Congress Ministries resigned in protest
all major Congress leaders the very next day. because the then Viceroy Lord Linlithgow
Gandhi, Nehru, Patel, etc. were all behind the (1936-1944) action of declared India to be
bars. This left the movement in the hands of the belligerent in WW-II without even consulting
younger leaders like Jayaprakash Narayan and the Indian people.
Ram Manohar Lohia, besides many regional 60. (b) Sub-Theme: Quit India Movement
leaders also rose up to lead this movement in The failure of the Cripps Mission to resolve
their respective regions. the constitutional deadlock made it clear to the
Option (c) is correct: The Quit India nationalists that any silence would amount to
movement was the spontaneous participation accepting the right of the British to decide the
of the masses compared to the other Gandhian fate of Indians without consulting them.
movements like non-cooperation and civil z Quit India Movement was launched in
disobedience. response to Mahatma Gandhi’s national
Option (d) is correct: Quit India Movement call for satyagraha. The All-India Congress
did not attract the labour class in general. Committee proclaimed a mass protest
NOTE: Here we have to identify the option demanding what Gandhiji called “an orderly
which is not true about the QIM. British withdrawal” from India.
58. (a) Sub-Theme: Quit India Movement z The Quit India resolution was ratified
at the Congress meeting at Gowalia Tank,
Option (a) is correct: Usha Sharma started
Bombay on August 8, 1942.
an underground Radio to give fuel to the
movement. Hence, she was well-known for 61. (c) Sub-Theme: Last phase of Indian Freedom
underground activities during the Quit India Struggle/Boundary Commission/Mountbatten
Movement. Plan/Transfer of Power
Option (b) is incorrect: Second Round Table z In the event of Partition, a boundary
Conference was held in London between commission led by Sir Cyril Radcliffe
September 7, 1931 to December 1, 1931 was established which was to delimit the
and Sarojini Naidu participated to represent boundaries between India and Pakistan.
women. Boundary Commission, consultative
Option (c) is incorrect: Usha Mehta was part committee created in July 1947 to
of a small group which ran the Congress radio. recommend how the Punjab and Bengal
She started anUnderground Radio in Bombay regions of the Indian subcontinent were to be
whereas Captain Lakshmi Sahgal was the divided between India and Pakistan shortly
before each was to become independent
commander of the Jhansi Regiment of INA.
from Britain.
Option (d) is incorrect: She was not a part of
the Interim Government. 62. (d) Sub-Theme: Congress Socialist Party
NOTE: It is always advisable to read in detail Statement 1 is incorrect: On May 17, 1934,
about the personalities involved in various at Patna, with Acharya Narendra Dev as
movements. If we observe PYQs carefully, in president and Jayaprakash Narayan (JP) as
every alternate year, there are personality general secretary, the Congress Socialist
based questions. Party (CSP) was founded within the Congress.
The CSP advocated decentralised socialism
59. (d) Sub-Theme: Resignation of Congress Ministries in which co-operatives, trade unions,
The Congress Ministries: In the early 1937, independent farmers, and local authorities
elections to provincial assemblies were would hold a substantial share of the economic
held and Congress Ministries were formed power. It did not advocate
in Bombay, Madras, Central Provinces, Statement 2 is incorrect: Their focus was to
United Provinces, Bihar, Orissa, Assam, attain independence and believed in socialism
and NWFP. By 1939, there were internal through nationalism. The CSP promoted a
strifes, opportunism and hunger for power had decentralised socialist system in which local
started surfacing among Congressmen, yet they authorities, cooperatives, trade unions, and

68 UPSC Prelims PYQs


independent farmers would control a sizable vest in the provinces of British India. Thus,
portion of the economic power. Hence, it did the Cabinet Mission plan proposed a weak
not aim to establish the dictatorship of the Centre with Provincial autonomy, essentially
proletariat. proposing a federal structure of government.
Statement 3 is incorrect: As secularists, Statements 2 and 3 are incorrect: The plan
they hoped to transcend communal divisions did not include the enlargement of the powers
through class solidarity. Many people, like of the Indian Court nor provided for more
Narendra Deva and Basawon Singh (Sinha), Indians in the ICS.
favoured a democratic socialism that was Objective of the Cabinet Mission:
separate from both reformist social democracy z To resolve political deadlock between the
and Marxism. Indian National Congress and the All-India
63. (b) Sub-Theme: Phases of Mahatma Gandhi/ Muslim League.
SatyagrahaMovements/Salt Satyagraha
z The Congress party wanted to obtain a
Civil Disobedience Movement - The Salt strong central government with more
Satyagraha and other upsurges: powers as compared to state governments.
Dandi March (March 12- April 6, 1930):
z The Dandi march was led by Gandhi and z All India Muslim League under Jinnah
began at Sabarmati Ashram and ended close wanted to keep India united but with political
to the Dandi village’s coastal location; it safeguards provided to Muslims such as
was also known as Salt Satyagraha. ‘guarantee’ of ‘parity’ in the legislatures.
z He reached Dandi on April 6, 1930, and z On 16 May 1946, this plan was announced
broke the salt law by collecting salt from and preceded by the Shimla Conference of
the beach. 1945.
Satyagraha at Different Places: z Recommendation:
z Tamil Nadu: C Rajagopalachari organised a  The unity of India had to be retained
march from Tiruchirapalli to Vedaranniyam  It proposed a very loose union of all
on Tanjore coast.
the Indian territories under a centre
z Malabar: K. Kelappan known for Vailkom that would control merely defence, the
Satyagraha organised salt marches. Foreign Affairs and the Communication.
z Orissa: Gopal Bandhu Chaudhuri The Union would have the powers
organised in Balasore, Cuttack etc. necessary to raise the finances to
z Bihar: Non chowkidari tax was imposed. manage these subjects.
z Peshawar: Badshah khan/khan Abdul  All subjects other than Union subjects
Gaffar Khan(Frontier Gandhi) who and residuary power would vest in
organised Khudaikhidmatgars (red shirts) the provinces of British India. Thus,
organised a powerful march. the Cabinet Mission plan proposed a
z Dharasana: Sarojini Naidu led the weak Centre with Provincial autonomy,
campaign. essentially proposing a federal
z Nagaland: Rani Gaidinliu at the age of structure of government.
13 years raised the banner against the British.  The Princely Legislatures would then
64. (a) Sub-Theme: Cabinet Mission elect a Constituent Assembly or a
Statement 1 is correct: On 22nd January Constitution making body with each
1946, the decision to send Cabinet Mission province being allotted a specified
was taken and on 19th February 1946, the number of seats proportionate to its
British PM C.R Attlee Government announced population.
in the House of Lords about the mission and  The proposed Constituent Assembly was
the plan to quit India. A high-powered mission
to consist of 292 members from British
of three British Cabinet members- Lord
India and 93 from Indian States.
Pethick-Lawrence(the Secretary of State
 The Mission proposed an immediate
for India), Sir Stafford Cripps(President of
the Board of Trade) and A. V. Alexander(the formation of Interim Government at the
First Lord of the Admiralty) reached Delhi on centre, enjoying the support of major
24th March 1946. All subjects other than political parties and with the Indians
Union subjects and residuary power would holding all the portfolios.

Modern History 69
65. (d) Sub-Theme: Cripps Mission Non-Cooperation Movement against the
Cripps Mission (1942): Japanese Invasion.
z In March 1942, a mission headed by z The resolution was accepted in the CWC
Stafford Cripps was sent to India with meeting in Wardha on July 14, 1942.
constitutional proposals to seek Indian Royal Indian Naval Mutiny (1946):
support for World War II. z On February 18, 1946, a mutiny started
z Proposals: among the ratings of HMIS Talwar due
 Indian union with a Dominion status to poor pay, inadequate food, and racial
would be set up and it would be free to discrimination.
decide its relation with the UN and the z The Muslim League, Congress, and CPI’s red
Commonwealth. flags were raised.
 After the war, the Constituent Assembly z The intervention of Sardar Patel put an end
would be formed with members from to the rebellion. The mutineers faced trials
Provinces (elected through proportional despite promises.
representation) and Princely states z This led to Violence in Bombay, with the loss
(nominated). of 200 lives.
z Conditions of accepting the New z Significance: On February 19, 1946, a
Constitution: Cabinet Mission was sent to India.
 Any Province not willing to join the
Union can have a separate Constitution NOTE: this question underscores the
and form a separate union. importance of understanding chain of
events and remembering relative timeline.
 New Constitution-making Body and the
In 2021, UPSC asked what happened on the
Government would negotiate a treaty
8th Aug 1942. So it is advisable to remember
for Transfer of Power.
the chronology of important events.
66. (c) Sub-Theme: Last phase of Indian Freedom 67. (b) Statement 1 is correct: During the early 1900s,
Struggle/RIN Movement Mahatma Gandhi was instrumental in getting
Second Round Table Conference (1931): the indentured labour system abolished in
z In London, between September 7, 1931 to the British Empire. He also supported Manilal
December 1, 1931. Doctor’s campaign to get a resolution passed
z Result of the Gandhi-Irwin Pact. in the INC meeting condemning the indentured
z Gandhi was the sole representative of the labour system in all British territories.
INC. Statement 2 is incorrect: Lord Chelmsford,
z Madan Mohan Malviya and A. Rangaswami the then Viceroy of India, invited Gandhi to
Iyengar were also there. Delhi at a War Conference. In order to gain the
z Muslim league represented by Aga Khan III, trust of the empire, Gandhi agreed to move
Maulana Mohammad Jinnah. people to enlist in the army for WW-I.
Statement 3 is correct: Indian National
z Depressed classes by B. R. Ambedkar,
Congress was declared illegal upon breaking of
Rettamalai Srinivasan, Sardar Sampuran
salt law. But this did not deter the satyagrahis
Singh etc.
who continued the movement.
z Sarojini Naidu participated to represent
68. (b) Sub-Theme: Phases of Mahatma Gandhi/Gandhi-
women.
Irwin Pact
z Lord Willingdon was the viceroy of India.
(UPSC didn’t give any answer in official key)
z A Communal Award for representing Possible ans (b)
minorities in India by providing for separate Statement 1 is correct: The Gandhi–Irwin
electorates for minority communities. Pact was a political agreement signed by
Gandhi was against this. Mahatma Gandhi and Lord Irwin, Viceroy of
z The government refused Indian demands India, on 5 March 1931 before the Second
and talks failed. Round Table Conference in London, where
Quit India Movement/August Revolution the Indian National Congress was invited to
(1942): participate.
z After the failure of Cripps’s Mission, Statement 2 is correct: Withdrawal of all
a Resolution was framed by Gandhiji ordinances issued by the Government of India
for British withdrawal and non-violent imposing curbs on the activities of the Indian

70 UPSC Prelims PYQs


National Congress was agreed by the Viceroy. z By holding open trials in the Red Fort,
Statement 3 is incorrect: The demand for a Claude Auchinleck, the commander-in-chief
public enquiry of excesses committed by police of the British-Indian army, hoped to sway
during the civil disobedience movement was public opinion against the INA.
rejected by the Viceroy. z Major General Shah Nawaz Khan,
Statement 4 is correct: Release of all political Colonel Prem Kumar Sahgal and Colonel
prisoners, except those guilty of violence was
Gurbaksh Singh Dhillon were three of the
also agreed.
senior-most officers of INA and trusted
NOTE: if we read Ncert, statement 1 is colleagues of Netaji.
general and common. Securing participation z After considerable demonstrations and
by the Indian National Congress in the
unrest in India, they were forced to be
Second Round Table Conference was an
released after the British court-martialed
important agenda of pact inter-alia. We also
them at Red Fort in 1945 and sentenced
know that viceroy rejected commutation of
the death sentence of Bhagat Singh, Rajguru them to death.
and Sukhdev to a life sentence, which was z Congress leader and the country’s first
criticised by several leaders. This underlines Prime Minister, Jawaharlal Nehru, also got
that prisoners with violence charges on board the INA officers’ legal defence
were not released. This reasoning makes team, along with party colleague Bhulabhai
statement 4 correct. Option (b) i.e 1, 2 and Desai and barrister Tej Bahadur Sapru.
4 only is the only option with statement 1
and 4 in it !! NOTE: As we have already discussed,
the trend of personality based questions
69. (a) Sub-Theme: Quit India Movement
continues !!
Option (a) is correct: Quit India Resolution was
ratified in a Congress meeting at Gowalia Tank 71. (a) Sub-Theme: Reforms in Education during the
on 8th August 1942. Apart from QIM, other British Time
resolutions involved the Civil Disobedience Wood’s Dispatch (1854):
Movement against British Rule. Mantra of “Do z Magna Carta of English Education in India.
or Die” was adopted in this Session by Gandhiji. z Government to take up the responsibility for
Option (b) is incorrect: More Indians were the Education of masses.
added to Viceroy’s executive council by the
z Hierarchy of Educational Institutions
Government of India Act 1919. First time,
an Indian was added in this council by the z Emphasise on Female, Vocational
Government of India act 1909. Education and teachers’ training.
Option (c) is incorrect: The Congress z Secular Education in Government
Ministries resigned from seven provinces in Institutions.
1939 in protest because the then Viceroy Lord z Grants-in-aid to encourage private
Linlithgow’s (1936-1944) action declared India education.
to be belligerent in the WW-II without even z Recommended English as the Medium
consulting the Indian people.
of Instruction in higher studies and
Option (d) is incorrect: Though Cripps
vernaculars in school level.
proposed a dominion status to India and the
failure of the cripps mission resulted in a z Universities with affiliations in three
nationwide Quit India Movement, this was not presidency towns of Calcutta, Bombay,
related to 8 August 1942. and Madras.
70. (d) Sub-Theme: Indian National Army/Netaji Statement 1 and 2 are correct: Grants-in-aid
Subhash Chandra Bose/Red Fort Trial to encourage private education and affiliating
Indian National Army (INA) Trials/ Red Fort universities in the presidency towns of Calcutta,
Trials: Bombay and Madras were recommended in the
z Between November 1945 and May 1946, Wood’s Despatch.
the INA officers were brought before a court Statement 3 is incorrect: The Woods Dispatch
martial at the Red Fort in Delhi. At the Red recommended English as the Medium of
Fort in Delhi, about ten court-martials were Instruction in higher studies and vernaculars
held in public. in school level.

Modern History 71
English language or Anglicist as the medium
NOTE: Grants-in-Aid system and
of instruction and the rest were supporters
Establishment of universities was very
of oriental or classic language or Classicists
popular recommendations of Woods
as a medium of instruction.
Dispatch, so option 1 and 2 are correct.
However, if you look carefully at option z This is the famous Anglicist and Classicist
3, it talks about English as a medium controversy.
of instruction at all levels of education. z On February 2, 1835, Lord Macaulay, who
Dispatch was in 1854, at that time introducing served as head of the General Committee
English as a medium of instruction at all of Public Instruction, penned a minute in
levels sounds unconvincing. By applying which he stated his position on the issue.
this crude logic, and taking the risk of z Through his minute, Lord Macaulay
eliminating option 3, we can get the answer. emphasised the use of the English language
as a medium of instruction. He believed
72. (d) Sub-Theme: Reforms in Education during the
that English was the ideal language for
British Time instruction.
Statement 1 is correct: The Charter Act of Orientalist-Anglicist controversy:
1813 permitted Christian missionaries to
propagate English and preach their religion. z Exclusively suggested for
Statement 2 is correct: As a president of the spending on modern
General Committee of Public Instruction, studies.
Lord Macaulay wrote a minute, where he z Even the Anglicists were
Macaulay stressed the implementation of the divided over the question
English language as a medium of instruction Anglicists of medium of instruction,
through his minute. opinion where one faction was for
Statement 3 is correct: One faction of the English language as the
Anglicists was in favour of English language medium, while the other
as the medium, while the other faction was in faction was for Indian
favour of Indian languages (vernaculars). languages (vernaculars)
Charter Act of 1813: for the purpose.
z The idea of supporting educated Indians z In order to prepare
and promoting knowledge of contemporary students for careers,
sciences in the country was enshrined into Western sciences and
the Charter Act of 1813. Orientalists
literature should be
opinion
z The Act required the Company to yearly taught, with a focus on
approve one lakh rupees for this purpose. advancing traditional
Even this meagre sum, though, was not Indian education.
made accessible until 1823, mostly due to 73. (b) Sub-Theme: Reforms in Education during the
the disagreement over the most appropriate British Time
strategy for this investment. Statement 1 is incorrect: The Sanskrit
z The Charter Act of 1813 gave Christian College was established by Jonathan Duncan,
missionaries the right to spread the English the resident, at Benaras in 1791 for study of
language and practise their religion. Hindu law and philosophy.
General Committee on Public Instruction Statement 2 is correct: The Calcutta
Madrasah was established by Warren
(1823):
Hastings in 1781 for the study of Muslim law
z A “General Committee of Public
and related subjects.
Instruction” was established by the
Statement 3 is incorrect: Fort William
Governor-General-in-Council in 1823 with College was set up by Lord Richard Wellesley
the task of allocating one lakh rupees for in 1800 for training of civil servants of the
educational purposes. Company in languages and customs of Indians
z The General Committee of Public Instruction (closed in 1802).
also was not able to decide the medium 74. (d) Sub-Theme: Reforms in Education during the
of instruction by vote; due to lack of British Time
majority. Fort William College (1800):
z Out of ten members of which Lord Macaulay z Fort William College was founded on
was the president, five were supporters of 10 July 1800 in Calcutta, British India.

72 UPSC Prelims PYQs


z Fort William College was set up by Lord z It was signed by Madan Mohan Malviya
Richard Wellesley in 1800 on behalf of Gandhi, and B. R. Ambedkar
z It was set up for the training of civil servants signed it on behalf of the depressed classes.
of the Company in languages and customs z The background to the Poona Pact was the
of Indians. Communal Award of August 1932, which,
z The court of directors of the British East among other things, reserved 71 seats in
India Company was not in support of a the central legislature for the depressed
training college in Kolkata. classes.
z Then it got closed in 1802 and a separate z Gandhi, who opposed the Communal Award
and believed it was a British effort to divide
College was established in 1807 in England.
Hindus, started a fast unto death in order
NOTE: Question on Fort William College to have it overturned.
came in 2018. By simply reading solutions z He rejected the concept of a special
of that question, it was possible to attempt electorate for the depressed classes.
this question correctly, simply earning z However, the number of seats set aside for
2 marks !! This underscores the importance them in provincial legislatures was increased
of solving and analysing PYQs. to 147, and in the Central Legislature, they
75. (c) Sub-Theme: Reforms in Education during the now make up 18% of all seats.
British Time z The findings of the Indian Franchise
Ishwar Chandra Vidyasagar (1820-1891) Committee, also known as the Lothian
was appointed as the secretary of Bethune Committee, served as the foundation for
School/College established in 1849. Bethune the communal award.
School began as Hindu Female School in 1849 78. (c) Sub-Theme: Important Personalities
was renamed Bethune School in 1856. In 1856, Annie Besant 1847-1933:
the Government took charge of the Hindu z She fought for the freedom of thought,
Female School, later renamed Bethune School. secularism, women’s rights, birth control,
NOTE: Details of Annie Besant has already workers’ rights and Fabian socialism.
been covered in PYQs under Prominent z Following her encounter with Helena
Personalities in Modern History. Blavatsky in 1889, Besant became a
76. (b) Sub-Theme: Important Personalities Theosophist.
Statement 1 is incorrect: The Peasants and z In 1893, she made her first trip to India with
Workers Party of India was founded in 1947. the Theosophical Society.
The party was founded in Maharashtra by z She and Henry Steel Olcott co-founded the
Tulsidas Jadhav, Keshavrao Jedhe and others. Theosophical Society - Adyar.
Statement 2 and 3 are correct: Both The z She was the society’s president from 1907
Independent Labour Party (1936) and All to 1933.
India Scheduled Castes Federation (1942) z In Benares, Besant founded the Central
were established by Dr. B.R. Ambedkar. Hindu College (CHC).
77. (c) Sub-Theme: Phases of Mahatma Gandhi z She joined the Indian National Congress
Communal Award (1932): (INC) and demanded government action
z The Prime Minister of Britain Ramsay towards self-rule. She became the President
MacDonald, announced the Communal of the Calcutta Session of INC, AD 1917.
Award in August 1932 to pursue the British z In 1916, Besant and Bal Gangadhar Tilak
policy of divide and rule. founded the All-India Home Rule League.
z The Communal Award recognised the z She did not attend the AD 1920 Session at
depressed classes as minorities and Nagpur due to growing differences with
provided them separate electorates. Gandhiji as she felt that the Government of
India Act, 1919 was a means to free India.
z Earlier the British had already given separate
electorates to Muslims, Christians and Sikhs. z Newspapers: New India and Commonweal.
Poona Pact 1932: z She wrote the ‘Lotus Song’, an English
z B.R. Ambedkar negotiated the Poona Pact version of the Gita.
with Mahatma Gandhi in late September Statement 1 is correct: Annie Besant fought
1932. for the freedom of thought, secularism,

Modern History 73
women’s rights, birth control, workers’ rights 3. The United States of America: A Hindu’s
and Fabian socialism. In 1916, Annie Besant Impression, 1916.
and Tilak started the Home Rule Leagues at 4. The problem of National Education in
Madras and Belgaum respectively. India, 1920
Statement 2 is incorrect: The Theosophical 5. Unhappy India, 1928.
Society was founded by Madame H. P.
Blavatsky and Colonel Olcott in New York in 6. England’s Debt to India, 1917.
1875. In 1882, the headquarters of the Society 7. Autobiographical Writings
were established in Adyar, near Madras (now 8. He also wrote biographies of Mazzini,
Chennai) in India. Garibaldi, Shivaji, and Shrikrishna.
Statement 3 is correct: Annie Besant became 82. (b) Sub-Theme: Literary Work during Indian
the President of the INC in Calcutta Session, National Movement
1917.
Option (a) is incorrect: The Arya samaj was
79. (b) Sub-Theme: Important Personalities/ Important founded by the Dayananda Saraswati on
sessions of INC 7 April 1875.
Statement 1 is incorrect: The first woman Option (b) is correct: Neel Darpan is a
President of the Indian National Congress was Bengali play, written by Dinabandhu Mitra in
Annie Besant in the 1917 Calcutta Session. 1858–1859.
Sarojini Naidu became the first Indian woman Option (c) is incorrect: Anandamath the
President of INC in 1925 Kanpur Session. famous Bengali novel written by Bankim
Statement 2 is correct: Badruddin Tyabji Chandra Chatterjee in 1882.
became the first Muslim President of the
Option (d) is incorrect: Satyendranath
Indian National Congress in the 1887 Madras
Tagore was selected for Indian Civil Service in
Session.
June 1863. He was the first Indian to succeed
80. (b) Sub-Theme: Social Movement Organisations in the ICS Exam in 1863.
Pair 1 is correct: Raja Radhakanta Deb was
the first President of the British Indian NOTE: As we have mentioned earlier, a
Association while Debendranath Tagore was chronology of events is important. This is
its secretary. another question on the same theme, earlier
z Pair 2 is incorrect: In 1884 Madras questions were asked in 2017 and 2021.
Mahajan Sabha was established by 83. (d) Sub-heme: Important Organisations
M. Veeraraghavachariar, G. Subramania Pair 1 is correct: On 30 September, 1932,
Iyer and P. Anandacharlu. Mahatma Gandhi founded the All India
z Pair 3 is correct: Surendranath Banerjee Anti Untouchability League, to remove
and Anand Mohan Bose founded the untouchability in society.
Indian Association of Calcutta in 1876. Pair 2 is correct: Sahajanand Saraswati
81. (c) Sub-Theme: Important Personalities was the first President of the All India Kisan
Lala Lajpat Rai: Sabha.
z Popularly known as Punjab Kesari. A Pair 3 is correct: The self-Respect Movement
member of the Lal Bal Pal trio. was started by E.V. Ramaswamy Naicker in
Tamil Nadu in 1925 with the aim of destroying
z The INC’s president at the 1920 Calcutta
Special Session. the contemporary Hindu social order in its
totality and creating a new, rational society
z In 1921, he founded Servants of the People
without caste, religion and god.
Society, a non-profit welfare organisation,
in Lahore. 84. (d) Sub-Theme: Important Personalities/ Important
Organisations
z Lajpat Rai travelled to the US in 1917.
Pair 1 is correct: All India Liberal Federation
z Led a non-violent march in protest to
the Simon Commission. Brutally assaulted was founded by Surendra Nath Banarjea
by the police and later succumbed to his and some of its prominent leaders were Tej
injuries. Bahadur Sapru, V. S. Srinivasa Sastri and
M. R. Jayakar and Tej Bahadur Sapru were
z Along with founding Arya Gazette as its
the presidents of Indian National Liberal
editor, authored:
Federation.
1. The Story of My Deportation, 1908.
Pair 2 is correct: K.C. Neogy was a member of
2. Arya Samaj, 1915. the Constituent Assembly of India, a member

74 UPSC Prelims PYQs


of the first Cabinet of independent India and 87. (c) Sub-Theme: Major events/Major places of Indian
the chairman of the first Finance Commission Freedom Struggle
of India. Option (a) is incorrect: The Indian tricolour
Pair 3 is correct: P.C. Joshi was one of the was designed by Pingali Venkayya, who was a
early leaders of the communist movement in freedom fighter and was a follower of Mahatma
India, he was the first general secretary of Gandhi. Pingali Venkayya had designed the flag
the Communist Party of India from 1935–47. of India and presented it to Mahatma Gandhi
85. (a) Sub-Theme: Literary Work during Indian in 1921 during the session of the All India
National Movement Congress Committee in Vijayawada.
Statement 1 and 2 are correct: In Option (b) is incorrect: During the QIM in
overwhelming detail, the book “Desher Katha” Andhra the Provincial Congress Committee
summarised the work of M. G. Ranade and had issued a circular popularly known as the
D. Naoroji in a popular idiom and warned in its ‘Kurnool Circular’ as the police seized the
concluding chapter against the colonial state’s copy when they rode ‘Kurnool Congress Office.
“hypnotic conquest of the mind”. “Desher This was drafted by Kala Venkat Rao, on 29th
Katha” had an immense repercussion in Bengal, July 1942 and was sent for the approval of
captured the mind of young Bengal and assisted the Congress Working Committee through Dr.
more than anything else in the preparation of Pattabhi Sitaramaiah, a member of the working
the Swadeshi Movement. committee.
Statement 3 is incorrect: Deuskar uses the Option (c) is correct: Rabindranath Tagore
word ‘desh’ in the context of the whole country. translated the National Anthem from Bengali
86. (a) Sub-Theme: Literary Work during Indian to English here.
National Movement Option (d) is incorrect: The Theosophical
Society was founded by Madame H. P.
Gopal Baba Walangkar (1840-1900):
Blavatsky and Colonel Olcott in New York in
z He was also known as Gopal Krishna, born
1875. In 1882, the headquarters of the Society
into a family of the untouchable Mahar caste
were established in Adyar, near Madras (now
in Maharashtra.
Chennai) in India.
z Gopal Baba Walangkar is an early example
88. (c) Sub-Theme: Important Personalities/Phases of
of a social activist working to untrammel
Mahatma Gandhi
the untouchable people of India from their
Quit India Movement planned an all-out
historic socio-economic oppression and he
campaign to compel British withdrawal from
is generally considered to be the pioneer of
India, after the failure of the Cripps Mission
that movement.
to reach a compromise. At the historic August
z He also developed a racial theory to explain meeting at Gowalia Tank in Bombay, Gandhi
the oppression and also published the first proclaimed his mantra—’do or die’. He was
journal targeted at the untouchable people. arrested on August 9, 1942. He undertook a
z The first publication to have the untouchable 21-day fast in February 1943 to protest against
people as its target readership was the Government actions against Indians involved
Walangkar’s Vital-Vidhvansak (Destroyer in the movement. Mahatma Gandhi wrote
of Brahmanical or Ceremonial Pollution), “Songs of Prison” during his imprisonment
which he started publishing in 1888. in Yerwada Jail, Poona. He translated hymns
z He also wrote articles for Marathi-language from the Upanishads and poems by Indian
newspapers such as Sudharak and saint poets into English and that collection was
Deenbandhu, as well as composed couplets published as Songs from the prison.
in Marathi that were intended to inspire the 89. (c) Sub-Theme: Constitutional Development
people. The idea of a constituent assembly was put
z In 1889, he published Vital Viduvansan forward for the first time by MN Roy. The
(Annihilation of Ceremonial Pollution), Indian National Congress (INC) first formally
which protested the position of untouchables requested a constituent assembly to draft an
in society and raised consciousness Indian constitution in 1935. Representatives
regarding what those people should expect. of each community were to be elected by
z He addressed this pamphlet, which was members of that community in the provincial
crafted as a collection of 26 questions, to legislative assembly and voting was to be by
the elites of Maharashtrian society. the method of proportional representation

Modern History 75
by means of single transferable vote.  292 seats allocated for British India were
Composition of the Constituent Assembly: to be from eleven governor’s provinces
z C.A. constituted in Nov 1946 under CMP and four from Chief commissioner’s
(Cabinet Mission Plan). provinces.
z Seats allotted as per population proportion.  Seats were allocated based on proportion
z Voting using a single transferable vote and to their respective population.
the proportional representation system  Muslims, Sikhs, and general voters were
when a province is represented. to determine which of the three major
populations in each British province
z Mahatma Gandhi was not part of C.A.
would hold the seats up for election in
z Strength of the assembly: the provincial legislative assembly.
 Total strength of the assembly: 389  The heads of these princely states
 296 seats for British India and 93 seats were supposed to nominate the
to princely states representatives.

v v v

76 UPSC Prelims PYQs

You might also like